CARDIAC QUIZ

Ace your homework & exams now with Quizwiz!

The reduction of LDL may be seen as early as ____ weeks after the initiation of statin drugs. 1 2 4 8

2

The nurse is assessing the preprinted Medication Administration Record (MAR) for a client admitted with angina. Which medication order should the nurse discuss with the pharmacist? 1. The 1130 regular insulin order. 2. The 0800 Glucophage order. 3. The 0900 Lipitor order. 4. The 2100 nitroglycerin order.

3 Lipitor should be administered in the evening (not at 0900) so that it will enhance the enzyme that works in the gastrointestinal system to help elimi- nate cholesterol. The nurse should notify the pharmacist and request a change in the time of administration.

The nurse is teaching the client diagnosed with angina about sublingual nitroglycerin (NTG), a coronary vasodilator. Which statement indicates the client needs more medication teaching? 1. "I will always carry my nitroglycerin in a dark-colored bottle." 2. "If I have chest pain, I will put a tablet underneath my tongue." 3. "If my pain is not relieved with one tablet, I will get medical help." 4. "I should expect to get a headache after taking my nitroglycerin."

3 The client should put one tablet under the tongue every 5 minutes and, if the chest pain is not relieved after taking three tablets, the client should seek medical attention. This statement indi- cates the client needs more teaching about the medication.

The nurse is preparing to administer medication to the following clients. Which medication should the nurse question administering? 1. The biguanide metformin (Glucophage) to a client with type 1 diabetes who is receiving insulin. 2. The loop diuretic bumetanide (Bumex) to a client diagnosed with essential hypertension. 3. The biologic response modifier erythropoietin (Procrit) to a client diagnosed with end-stage renal failure. 4. The central-acting alpha agonist clonidine (Catapres) to a client diagnosed with heart failure.

4 The nurse would question administering Catapres to a client with decreased car- diac output (heart failure), because this medication acts within the brain stem to suppress sympathetic outflow to the heart and blood vessels. The result is vasodilation and reduced cardiac output, both of which lower blood pressure.

While preparing a patient for discharge, which of the following statements should the nurse include in instructions regarding his new prescription for furosemide (Lasix)? A."Be sure to change your position slowly and rise slowly after sitting or lying to prevent dizziness and possible fainting." B."Avoid foods high in potassium, such as bananas, oranges, fresh vegetables and dates." C."If you experience weight gain such as 5 or more pounds a week, be sure to tell your physician during your next routine visit." D."Keep a monthly journal or log of your weight."

A Orthostatic hypotension is a possible problem with diuretic therapy because of the diuretic-induced fluid volume loss.

Question 11 Furosemide is administered intravenously to a client with HF. How soon after administration should the nurse begin to see evidence of the drugs desired effect? A 5 to 10 minutes B 30 to 60 minutes C 2 to 4 hours D 6 to 8 hours

A After IV injection of furosemide, diuresis normally begins in about 5 minutes and reaches its peak within about 30 minutes. Medication effects last 2 to 4 hours.

What should patients taking thiazide diuretics do when they "feel worse" or are uncomfortable with their medication? A Call the prescriber to report discomfort. B Hold the medication until they feel better. C Discontinue the medication and request change of medication. D Skip taking the current dose and resume medication with the next dose.

A Patients are recommended to report drug-related discomfort to the prescriber for appropriate follow-up action.

A nurse knows that which diuretic is most frequently combined with an antihypertensive drug? A chlorthalidone B hydrochlorothiazide C bendeoflumethiazide D potassium-soaring diuretic

B

All of the following are correct regarding Bumex except for:________________ A it is a diuretic B should be taken in the afternoon C it should be taken in the morning D may cause potassium depletion

B

Homocysteine is a protein in the blood that has been linked with cardio disease and stroke. What other negative action may it promote? A flushing of the skin B loss of blood vessel elasticity C Photosensitivity and sunburn D Lowering of LDL levels

B

After the nurse teaches the patient about the use of carvedilol (Coreg) in preventing anginal episodes, which statement by a patient indicates that the teaching has been effective? A "Carvedilol will help my heart muscle work harder." B "It is important not to suddenly stop taking the carvedilol." C "I can expect to feel short of breath when taking carvedilol." D "Carvedilol will increase the blood flow to my heart muscle."

B Patients who have been taking β-adrenergic blockers can develop intense and frequent angina if the medication is suddenly discontinued. Carvedilol (Coreg) decreases myocardial contractility.

Amiodarone HCL is a(n):___________________drug? A respiratory B endocrine C cardiac D gastrointestinal

C

This herb is known to cause intermittent claudication in patients. A ginger B Ginseng C Ginkgo D Goldenseal

C

Which elevated apolipoprotein can be an indication of risk for coronary heart disease? A Apo-A-1 B Apo-A-2 C Apo B-100 D Apo C-4

C

Which adverse reaction to loop diuretic therapy requires immediate nursing intervention? A Dizziness B Thirst C Hearing loss D Weight gain

C Hearing loss in patients taking loop diuretics indicates ototoxicity and must be immediately reported to the prescriber.

In preparation for discharge, the nurse teaches a patient with chronic stable angina how to use the prescribed short-acting and long-acting nitrates. Which patient statement indicates that the teaching has been effective? A "I will check my pulse rate before I take any nitroglycerin tablets." B "I will put the nitroglycerin patch on as soon as I get any chest pain." C "I will stop what I am doing and sit down before I put the nitroglycerin under my tongue." D"I will be sure to remove the nitroglycerin patch before taking any sublingual nitroglycerin."

C The patient should sit down before taking the nitroglycerin to decrease cardiac workload and prevent orthostatic hypotension.

Which drug/herbal history of a patient will prompt the nurse to report a possible drug-drug interaction in a patient taking a thiazide diuretic? A Furosemide B Spironolactone C Digoxin D Propranolol

C Thiazide and loop diuretics cause hypokalemia, which enhances the action of digoxin. If digoxin is taken with thiazide and loop diuretics, this can result in digoxin toxicity.

What intervention will the nurse perform when monitoring a patient receiving triamterene (Dyrenium)? A Assess urinary output every other day. B Monitor for side effect of hypoglycemia. C Assess potassium levels. D Monitor for hypernatremia.

C Triamterene (Dyrenium) is a potassium-sparing diuretic. The nurse should monitor potassium for potential hyperkalemia.

A client has been taking furosemide (Lasix) for the past week. The nurse recognizes which finding may indicate the client is experiencing a negative side effect from the medication? A Weight gain of 5 pounds B Edema of the ankles C Gastric irritability D Decreased appetite

D Lasix causes a loss of potassium if a supplement is not taken. Signs and symptoms of hypokalemia include anorexia, fatigue, nausea, decreased GI motility, muscle weakness, dysrhythmias.

Which side effect of rosuvastatin (Crestor) is reported to the provider immediately? A Constipation B Abdominal pain C Muscle pain D Nausea

If the patient taking rosuvastatin (Crestor) complains of muscle pain or tenderness, it should be reported immediately. This could indicate the more serious side effect of rhabdomyolysis.

How are statin drugs administered? IV SubQ PO IM

PO

A client is taking digoxin (Lanoxin) 0.25 mg and furosemide (Lasix) 40 mg. When the nurse enters the room, the client states, "There are yellow halos around the lights." Which action will the nurse take? a. Evaluate digoxin levels. b. Withhold the furosemide c. Administer potassium. d. Document the findings and reassess in 1 hour.

a

A client is to be discharged home with a transdermal nitroglycerin patch. Which instruction will the nurse include in the client's teaching plan? a. "Apply the patch to a nonhairy area of the upper torso or arm." b. "Apply the patch to the same site each day." c. "If you have a headache, remove the patch for 4 hours and then reapply." d. "If you have chest pain, apply a second patch next to the first patch."a

a

The client is also taking a diuretic that decreases her potassium level. The nurse expects that a low potassium level (hypokalemia) could have what effect on the digoxin? a. Increase the serum digoxin sensitivity level b. Decrease the serum digoxin sensitivity level c. Not have any effect on the serum digoxin sensitivity level d. Cause a low average serum digoxin sensitivity level

a

The client's serum digoxin level is 3.0 ng/mL. What does the nurse know about this serum digoxin level? a. It is in the high (elevated) range. b. It is in the low (decreased) range. c. It is within the normal range. d. It is in the low average range.

a

When a client is taking ezetimibe (Zetia), she asks the nurse how it works. The nurse should explain that Zetia does what? a. Inhibits absorption of dietary cholesterol in the intestines. b. Binds with bile acids in the intestines to reduce LDL levels. c. Inhibits HMG-CoA reductase, which is necessary for cholesterol production in the liver. d. Forms insoluble complexes and reduces circulating cholesterol in bloo

a

A patient is to be discharged with a transdermal nitroglycerin patch. Which instruction will the nurse include in the patient's teaching plan? A "Apply the patch to a non-hairy area of the upper torso or arm." B "Apply the patch to the same site each day." C "If you have a headache, remove the patch for 4 hours and then reapply." D "If you have chest pain, apply a second patch next to the first patch."

a A nitroglycerin patch should be applied to a non-hairy area for the best and most consistent absorption rates. Sites should be rotated to prevent skin irritation. The drug should be continued if headache occurs, as tolerance will develop. Sublingual nitroglycerin should be used to treat chest pain.

A client with heart failure is digitalized and placed on a maintenance dose of digoxin (Lanoxin) 0.25 mg by mouth daily. What responses does the nurse expect the client to exhibit when a therapeutic effect of digoxin is achieved? a. Diuresis and decreased pulse rate c. Increased blood pressure and weight loss d. Regular pulse rhythm and stable fluid balance e. Corrected heart murmur and decreased pulse pressure

a Digoxin slows the heart rate, which is reflected in a slowing of the pulse; it also increases kidney perfusion, which promotes urine formation, resulting in diuresis and decreased edema.

One week after being hospitalized for an acute myocardial infarction, a client reports loss of appetite and feeling nauseated. Which of the client's prescribed medications should be withheld and the health care provider notified? a. Digoxin (Lanoxin) b. Furosemide (Lasix) c. Propranolol (Inderal) d. Spironolactone (Aldactone)

a Toxic levels of digoxin stimulate the medullary chemoreceptor trigger zone, resulting in anorexia, nausea, and vomiting.

A client is prescribed a noncardioselective beta1 blocker. What nursing intervention is a priority for this client? a. Assessment of blood glucose levels b. Respiratory assessment c. Orthostatic blood pressure assessment d. Teaching about potential tachycardia

b

A client is prescribed gemfibrozil (Lopid) for treatment of hyperlipidemia type IV. What is important for the nurse to teach the client? a. "Take aspirin before the medication if you experience facial flushing." b. "You may experience headaches with this medication." c. "You will need to have weekly blood drawn to assess for hyperkalemia." d. "Cholesterol levels will need to be assessed daily for one week."

b

A client is taking hydrochlorothiazide 50 mg/day and digoxin 0.25 mg/day. What type of electrolyte imbalance does the nurse expect to occur? a. Hypocalcemia b. Hypokalemia c. Hyperkalemia d. Hypermagnesemia

b

A client receiving intravenous nitroglycerin at 20 mcg/min complains of dizziness. Nursing assessment reveals a blood pressure of 85/40 mm Hg, heart rate of 110 beats/min, and respiratory rate of 16 breaths/min. What is the nurse's priority action? a. Assess the client's lung sounds. b. Decrease the intravenous nitroglycerin by 10 mcg/min. c. Stop the nitroglycerin infusion for 1 hour, and then restart. d. Recheck the client's vital signs in 15 minutes but continue the infusion.

b

The client asks the nurse how nitroglycerin should be stored while traveling. What is the nurse's best response? a. "You can protect it from heat by placing the bottle in an ice chest." b. "It's best to keep it in its original container away from heat and light." c. "You can put a few tablets in a resealable bag and carry it in your pocket." d. "It's best to lock them in the glove compartment to keep them away from heat and light."

b

A client with a history of type 1 diabetes is diagnosed with heart failure. Digoxin (Lanoxin) is prescribed. When administering the medication, the nurse should: a. Administer the medication with 8 ounces of orange juice b. Monitor the client for atrial fibrillation and first-degree heart block c. Administer the digoxin one hour after the client's morning insulin d. Withhold the medication if the apical pulse rate is greater than 60 beats/min

b The speed of conduction is decreased when digoxin is given, and this can result in premature beats, atrial fibrillation, and first-degree heart block.

What must the nurse monitor when titrating intravenous nitroglycerin for a client? (Select all that apply.) a. Continuous oxygen saturation b. Continuous blood pressures c. Hourly ECGs d. Presence of chest pain e. Serum nitroglycerin levels f. Visual acuity

b c d

A nurse is caring for a client taking cholestyramine (Questran). The client is complaining of constipation. What will the nurse do? a. Call the health care provider to change the medication. b. Tell the client to skip a dose of the medication. c. Have the client increase fluids and fiber in his diet. d. Administer an enema to the client.

c

The beta blocker acebutolol (Sectral) is prescribed for dysrhythmias. The nurse knows that what is the primary purpose of the drug? a. To increase the beta1 and beta2 receptors in the cardiac tissues b. To increase the flow of oxygen to the cardiac tissues c. To block the beta1-adrenergic receptors in the cardiac tissues d. To block the beta2-adrenergic receptors in the cardiac tissues

c

What would cause the same client's electrolyte imbalance? a. High dose of digoxin b. Digoxin taken daily c. Hydrochlorothiazide d. Low dose of hydrochlorothiaizde

c

A client with a history of heart failure and hypertension is admitted with reports of syncope. Which prescribed medication should the nurse prepare to administer based on the ECG rhythm strip image? a. Digoxin (Lanoxin) b. Enalapril (Vasotec) c. Atropine d. Metoprolol (Lopressor)

c Atropine, an anticholinergic that increases the heart rate, is administered when the heart rate is so slow that it causes symptoms.

A nurse is monitoring a client with angina for therapeutic effects of nitroglycerin. Which assessment finding indicates that the nitroglycerin has been effective? a. Blood pressure 120/80 mm Hg b. Heart rate 70 beats per minute c. ECG without evidence of ST changes d. Client stating that pain is 0 out of 10

d

Statin users should avoid ______ and _____ juice, as it can decrease the metabolism of statins, which leads to increased levels of statin in the body.

grapefruits

The client taking digoxin (Lanoxin), a cardiac glycoside, has a serum digoxin level of 4.2 ng/mL. Which medication should the nurse anticipate the HCP prescribing? 1. The digitalis binder Fab antibody fragments (Digibind). 2. The loop diuretic furosemide (Lasix). 3. The HCP will not prescribe any medications. 4. The cardiac glycoside digoxin (Lanoxin).

1 When digoxin overdose is suspected, as it would be with a digoxin level of 4.2 ng/mL. Fab antibody fragments bind digoxin and prevent it from acting. The therapeutic range of digoxin is 0.5-1.2 ng/mL and toxic range is 2.0 ng/mL or higher.

The client on telemetry is showing multifocal premature ventricular contrinterventions. Which antidysrhythmic medication should the nurse administer? 1. Lidocaine. 2. Atropine. 3. Adenosine. 4. Epinephrine.

1 Lidocaine suppresses ventricular ectopy and is a first-line drug for the treatment of ventricular dysrhythmias.

The elderly client diagnosed with coronary artery disease has been taking aspirin daily for more than a year. Which data warrant notifying the health-care provider? 1. The client has lost 5 pounds in the last month. 2. The client has trouble hearing low tones. 3. The client reports having a funny taste in the mouth. 4. The client has hard, dark, tarry stools.

4 A complication of long-term aspirin use is gastric bleeding, which could result in dark, tarry stools. This data would warrant further intervention.

What is a priority nursing diagnosis for a patient taking an antihypertensive medication? A Alteration in cardiac output related to effects on the sympathetic nervous system B Knowledge deficit related to medication regimen C Fatigue related to side effects of medication D Alteration in comfort related to nonproductive cough

A Circulation is always a priority over fatigue, pain, and knowledge deficit.

What is the most significant adverse effect of spironolactone? A hyperkalemia B hypokalemia C Hypernatremia D Hyponateremia

A Spironolactone has potassium-sparing effects and is contraindicated in patients with hyperkalemia. Hyperkalemia is a risk of this drug.

Which nursing intervention will decrease the flushing reaction of niacin? A Administering niacin with an antacid B Administering aspirin 30 minutes before nicotinic acid C Administering diphenhydramine hydrochloride (Benadryl) with niacin D Applying cold compresses to the head and neck

B Administration of an antiinflammatory agent such as aspirin has been shown to decrease the flushing reaction associated with niacin. In addition, avoiding hot beverages, such as coffee, when taking niacin may also prevent flushing.

A patient receiving nitroprusside begun manifesting headache, distant heart sounds, imperceptible pulses, and shallow breathing. The nurse knows that these are signs of? A. Reflex tachycardia B. Hypothyroidism C. Cyanide toxicity D. Severe alteration of blood pressure

C

The nurse reviews the history for a patient taking atorvastatin (Lipitor). What will the nurse act on immediately? A The patient takes medications with grape juice. B The patient takes herbal therapy including kava kava. C The patient is on oral contraceptives. D The patient was started on penicillin for a respiratory infection.

C Atorvastatin (Lipitor) increases the estrogen levels of oral contraceptives. The patient's oral contraceptive may need to be altered.

A patient with diabetes mellitus and chronic stable angina has a new order for captopril (Capoten). The nurse should teach the patient that the primary purpose of captopril is to A lower heart rate. B control blood glucose levels. C prevent changes in heart muscle. D reduce the frequency of chest pain.

C The purpose for angiotensin-converting enzyme (ACE) inhibitors in patients with chronic stable angina who are at high risk for a cardiac event is to decrease ventricular remodeling. ACE inhibitors do not directly impact angina frequency, blood glucose, or heart rate.

The patient is taking atorvastatin (lipitor) 80 mg/day. The patients partner calls to report that the patient is feeling week and complaining of muscle pain. What severe side effect pf statins does the nurse suspect? A Stevens-Johnson syndrome B Pseudomembranous colitis C Gastric ulcers D Rhabdomyolsis

D

What is the standard level for LDL? A less than 250 mg/dL B less than 200 mg/dL C less than 150 mg/dL D less than 100 mg/dL

D

Which of the following is a is not an ACE inhibitor A Benazepril B Captopril C Lisonopril D Verapamil

D

Which action of a loop diuretic can potentially cause hypotension in patients taking this medication? A Tubular reabsorption of sodium and chloride B Water retention and potassium-sparing C Excess fluid loss and dehydration D Decreased blood volume and venous constriction

D Loss of water and sodium reduces blood volume. Loop diuretics do not cause smooth muscle constriction.

A nurse is monitoring a client who is taking Carvedilol (Coreg CR). Which of the following assessment made by the nurse would warrant a possible complication with the use of this medication? A Baseline blood pressure of 160/100 mm hg followed by a blood pressure of 120/70 mm hg after 3 doses. B Baseline heart rate of 97 bpm followed by a heart rate of 62 bpm after 3 doses. C Complaints of nightmares and insomnia. D Complaints of dyspnea.

D Complaints of dyspnea is a sign of bronchospasm which is one of the serious complication of beta blockers. Options A and B shows a decrease in the blood pressure and heart rate which are expected in this therapy. Option C is a side effect of this medication.

A nurse is monitoring a client who is taking Digoxin (Lanoxin). All of which are the side effects associated with the medication, except? A Anorexia. B Blurred vision C Diarrhea. D Tremors.

D Signs of digoxin toxicity are as follows, anorexia, nausea, vomiting, diarrhea, and blurred vision.

The nurse acknowledges that the first-line drug for treating this client's blood pressure might be which drug? a. Diuretic b. Alpha blocker c. ACE inhibitor d. Alpha/beta blocker

a

Which statement indicates that the client needs additional instruction about antihypertensive treatment? a. "I will check my blood pressure daily and take my medication when it is over 140/90." b. "I will include rest periods during the day to help me tolerate the fatigue my medicine may cause." c. "I will change my position slowly to prevent feeling dizzy." d. "I will not mow my lawn until I see how this medication makes me feel."

a

The nurse knows that which diuretic is most frequently combined with an antihypertensive drug? a. chlorthalidone b. hydrochlorothiazide c. bendroflumethiazide d. potassium-sparing diuretic

b

An older adult with cerebral arteriosclerosis is admitted with atrial fibrillation and is started on a continuous heparin infusion. What clinical finding enables the nurse to conclude that the anticoagulant therapy is effective? a. A reduction of confusion b.An activated partial thromboplastin (APTT) twice the usual value c. An absence of ecchymotic areas d. A decreased viscosity of the blood

b Desired anticoagulant effect is achieved when the activated partial thromboplastin time is 1.5 to 2 times normal. While anticoagulants help prevent thrombi that could block cerebral circulation, they do not increase cerebral perfusion, and so will not affect existing confusion.

A client who had a myocardial infarction receives 15 mg of morphine sulfate for chest pain. Fifteen minutes after receiving the drug, the client complains of feeling dizzy. What action should the nurse take? a. Determine if this is an allergic reaction b. Place the client in the supine position and take the vital signs c. Elevate the client's head and keep the extremities warm d. Tell the client that this is not a typical sensation after receiving morphine sulfate

b Vertigo is a symptom of hypotension, a side effect of morphine sulfate. The supine position increases venous return, increases cardiac output, and increases blood flow to the brain. Dizziness is a symptom of hypotension that is a side effect, not an allergic response, to morphine sulfate. Raising the client's head may aggravate dizziness. Dizziness is a typical side effect of morphine sulfate.

A client with acute pulmonary edema receives furosemide (Lasix). What assessment finding indicates that the intervention is working? a. Potassium level decreased from 4.5 to 3.5 mEq/L. b. Crackles auscultated in the bases. c. Lungs clear. d. Output 30 mL/hr.

c

A client with hyperaldosteronism is prescribed spironolactone (Aldactone). What assessment finding would the nurse evaluate as a positive outcome? a. Decreased potassium level b. Decreased crackles in the lung bases c. Decreased aldosterone d. Decreased ankle edema

c

A client with a history of arthritis has an acute episode of right ventricular heart failure and is receiving furosemide (Lasix). The health care provider lowers the client's usual dosage of aspirin. The client asks the nurse the reason for the lower dose. What is the nurse's best response? a. "Aspirin accelerates metabolism of furosemide and decreases the diuretic effect." b. "Aspirin in large doses after an acute stress episode increases the bleeding potential." c. "Competition for renal excretion sites by the drugs causes increased serum levels of aspirin." d. "Use of furosemide and aspirin concomitantly increases formation of uric acid crystals in the nephron."

c Because furosemide and aspirin compete for the same renal excretory sites, salicylate toxicity may occur even with lower dosages.

A client who had a myocardial infarction receives a prescription for a beta-blocker and a nitroglycerin patch. The nurse determines that the purpose of the nitroglycerin patch is to decrease the: a. Pulse rate, thereby strengthening cardiac contractility b. Cardiac output, thereby reducing the cardiac workload c. Preload of the heart, thereby reducing the cardiac workload d. Coronary artery lumens, thereby reducing peripheral resistance

c Nitroglycerin reduces cardiac workload by decreasing the preload of the heart by its vasodilating effect; it dilates coronary arteries, reduces myocardial ischemia, strengthens contractility, and increases efficiency of cardiac output.

Captopril (Capoten) has been ordered for a client. The nurse teaches the client that ACE inhibitors have which common side effects? a. Nausea and vomiting b. Dizziness and headaches c. Upset stomach d. Constant, irritating cough

d

The nurse is caring for a client with hypertension who is prescribed Clonidine transdermal preparation. What is the correct information to teach this client? a. Change the patch daily at the same time. b. Remove the patch before taking a shower or bath. c. Do not take other antihypertensive medications while on this patch. d. Get up slowly from a sitting to a standing position.

d

Which statement made by the client indicates understanding about discharge instructions on antihyperlipidemic medications? a. "Antihyperlipidemic medications will replace the other interventions I have been doing to try to decrease my cholesterol." b. "It is important to double my dose if I miss one in order to maintain therapeutic blood levels." c. "I will stop taking the medication if it causes nausea and vomiting." d. "I will continue my exercise program to help increase my high-density lipoprotein serum levels."

d

Sublingual nitroglycerin is prescribed for a client with a history of a myocardial infarction and atrial tachycardia. The nurse instructs the client about the prophylactic use of these tablets. The statement by the client that indicates the teaching was effective is, "I should: a. ... take the medicine three times a day." b. ... avoid activities that are too strenuous." c. ... be sure to take my pulse after I have exercised." d. ... take one tablet before attempting to climb two flights of stairs."

d The response about taking one tablet before attempting to climb two flights of stairs indicates that the client understands the nurse's teaching. Taking a nitroglycerin tablet before such an activity probably will prevent an episode of angina, which is an example of prophylactic use of a medication.

The nurse is administering 0900 medications to the following clients. Which client should the nurse question administering the medication? 1. The client receiving a calcium channel blocker who drank a glass of grapefruit juice. 2. The client receiving a beta blocker who has an apical pulse of 62 beats per minute. 3. The client receiving a nitroglycerin patch who has a blood pressure of 148/92. 4. The client receiving an antiplatelet medication who has a platelet count of 150,000.

1 The client receiving a calcium channel blocker (CCB) should avoid grapefruit juice because it can cause the CCB to rise to toxic levels. MEDICATION MEMORY JOGGER: Grapefruit juice can inhibit the metabolism of certain medications. Specifically, grapefruit juice inhibits cytochrome P450-3A4 found in the liver and the intestinal wall. The nurse should investigate any medications the client is taking if the client drinks grape- fruit juice.

The nurse is preparing to administer spironolactone (Aldactone), a potassium-sparing diuretic. Which priority intervention should the nurse implement? 1. Check the client's potassium level. 2. Monitor the client's urinary output. 3. Encourage consumption of potassium-rich foods. 4. Give the medication with food.

1 When preparing to administer a potassium-sparing diuretic, the nurse should check the potassium level because both hyperkalemia and hypokalemia can result in cardiac dysrhythmias that are life threatening. Therefore, checking potassium level is a priority nursing intervention.

The client in congestive heart failure (CHF) is prescribed milrinone lactate (Primacor), a phosphodiesterase inhibitor. Which priority intervention should the nurse implement? 1. Assess the client's respiratory status. 2. Monitor the client's telemetry strip. 3. Check the client's apical pulse rate. 4. Evaluate the brain natriuretic peptide (BNP).

2 Primacor inhibits the enzyme phospho-diesterase, thus promoting a positive inotropic response and vasodilatation. Severe cardiac dysrhythmias may result from this medication; therefore, the client's telemetry should be monitored.

A patient who is taking a thiazide diuretic has been admitted to the hospital. The nurse should notify the prescriber of which side effect? A Hyperuricemia B Hypokalemia C Hypoglycemia D Hyperammonemia

A Hyperuricemia is excess uric acid in the blood stream, and is a side effect of thiazide diuretics.

A nurse is caring for a patient who has been started on ibutilide (Corvert). Which assessment is a priority for this patient? A Blood pressure measurement B BUN and creatinine C ECG D Lung sounds

C Ibutilide (Corvert) is specifically indicated for treatment of recent-onset atrial fibrillation and flutter. It is important for the nurse to obtain an ECG to see if the patient has converted to sinus rhythm.

The nurse is caring for a patient with hypertension who is prescribed a clonidine transdermal patch. What is the correct information to teach this patient? A Change the patch daily at the same time. B Remove the patch before taking a shower or bath. C Do not take other antihypertensive medications while on this patch. D Get up slowly from a sitting to a standing position.

D This medication can cause dizziness. Patient safety is a priority. The patch is left on for 7 days and can be left on while bathing. This medication is often prescribed with other drugs.

Which of the following is a factor r/t hypertension. A hypovolemia B decreased CO C increased Preload D increased after load

D Blood pressure increases as vascular volume increases (preload). Blood pressure increases as cardiac output (heart rate X stroke volume) increases. Vasoconstriction causes increased afterload and increased blood pressure.

A student nurse asks her instructor, "Can you help me understand the difference between furosemide and thiazide diuretics?" How should the instructor respond? A "Furosemide can cause hypokalemia. A thiazide diuretic does not cause hypokalemia." B "Neither furosemide nor thiazide place a patient at risk for dehydration." C "Eating bananas is only indicated for patients taking furosemide. Patients on a thiazide diuretic do not benefit from potassium rich foods." D "Furosemide should not be added to another loop diuretic. A thiazide diuretic can be added to a loop diuretic."

D Furosemide administered in combination with another loop diuretic would be too potent. A thiazide may be added to a loop diuretic to potentiate diuresis.

The nurse acknowledges that which condition could occur when taking furosemide? a. Hypokalemia b. Hyperkalemia c. Hypoglycemia d. Hypermagnesemia

a

Which is a priority nursing diagnosis for a client taking an antihypertensive medication? a. Alteration in cardiac output related to effects on the sympathetic nervous system b. Knowledge deficit related to medication regimen c. Fatigue related to side effects of medication d. Alteration in comfort related to nonproductive cough

a

The nurse is caring for a client who is scheduled for an electrophysiology study (EPS) because of persistent ventricular tachycardia. Before the procedure the client is to receive a beta blocker. The client's response during the procedure that best indicates that the beta blocker is working effectively is: a. Decreased heart rate b. Decreased anxiety c. Reduced chest pain d. Increased blood pressure

a A decreased heart rate or sinus bradycardia is the expected response to a beta-blocker. Beta-blockers inhibit the activity of the sympathetic nervous system and of adrenergic hormones, decreasing the heart rate, conduction velocity, and workload of the heart. Beta blockers reduce blood pressure.

When a client first takes a nitrate, the nurse expects which symptom that often occurs? a. Nausea and vomiting b. Headaches c. Stomach cramps d. Irregular pulse rate

b

A client has a serum cholesterol level of 265 mg/dL, triglyceride level of 235 mg/dL, and LDL of 180 mg/dL. What do these serum levels indicate? a. Hypolipidemia b. Normolipidemia c. Hyperlipidemia d. Alipidemia

c

A nurse teaching a client who has diabetes mellitus and is taking hydrochlorothiazide 50 mg/day. The teaching should include the importance of monitoring which levels? a. Hemoglobin and hematocrit b. Blood urea nitrogen (BUN) c. Arterial blood gases d. Serum glucose (sugar)

d

The client diagnosed with a myocardial infarction is receiving thrombolytic therapy. Which data warrants immediate intervention by the nurse? 1. The client's telemetry has reperfusion dysrhythmias. 2. The client is oozing blood from the intravenous site. 3. The client is alert and oriented to date, time, and place. 4. The client has no signs of infiltration at the insertion site.

2 Any bleeding from the intravenous site, gums, rectum, or vagina should be reported to the HCP. The HCP may not be able to take intervention to prevent the bleeding during therapy, but it war- rants notifying the HCP.

The client being discharged after sustaining an acute myocardial infarction is prescribed the ACE inhibitor lisinopril (Zestril). Which instruction should the nurse include when teaching about this medication? 1. Instruct the client to monitor the blood pressure weekly. 2. Encourage the client to take medication on an empty stomach. 3. Discuss the need to rise slowly from lying to a standing position. 4. Teach the client to take the medication at night only.

3 This medication causes orthostatic hypotension, and the client should be instructed to rise slowly from lying to sitting to standing position to prevent falls and injury.

The client diagnosed with coronary artery disease is instructed to take 81 mg of aspirin ("baby aspirin," "children's aspirin" or "adult low-dose aspirin") daily. Which statement best describes the scientific rationale for prescribing this medication? 1. This medication will help thin the client's blood. 2. Daily aspirin will decrease the incidence of angina. 3. This medication will prevent platelet aggregation. 4. Baby aspirin will not cause gastric distress.

3 When a baby aspirin is taken daily, it helps prevent platelet aggregation, which, in turn, helps the blood pass through the narrowed arteries more easily.

The client calls the clinic and says, "I am having chest pain. I think I am having another heart attack." Which intervention should the nurse implement first? 1. Call 911 emergency medical services. 2. Instruct the client to take an aspirin. 3. Determine if the client is at home alone. 4. Ask if the client has any sublingual nitroglycerin.

4 Because the client has had one myocar- dial infarction, the client may have sub- lingual nitroglycerin in a pocket and can take it immediately. If the client does not have any on the body, then the nurse should determine if there is anyone in the home that can help the client.

16. The nurse is caring for the clients on the telemetry unit. Which medication should the nurse administer first? 1. The antiplatelet medication clopidogrel (Plavix) to the client with arterial occlusive disease. 2. The cardiac glycoside digoxin (Lanoxin) to the client diagnosed with congestive heart failure. 3. The iron dextran infusion to the client diagnosed with iron-deficiency anemia who has pale skin. 4. The antidysrhythmic amiodarone (Cordarone) to the client in ventricular bigeminy on the telemetry monitor.

4 Ventricular bigeminy is a life-threatening dysrhythmia that must be treated imme- diately to prevent cardiac arrest.

A patient who has recently started taking pravastatin (Pravachol) and niacin (Nicobid) reports the following symptoms to the nurse. Which is most important to communicate to the health care provider? A Generalized muscle aches and pains B Dizziness when changing positions quickly C Nausea when taking the drugs before eating D Flushing and pruritus after taking the medications

A Muscle aches and pains may indicate myopathy and rhabdomyolysis, which have caused acute kidney injury and death in some patients who have taken the statin medications.

A patient is taking digoxin (Lanoxin) 0.25 mg and furosemide (Lasix) 40 mg. The patient tells the nurse, "There are yellow halos around the lights." Which action will the nurse take? A Evaluate digoxin levels. B Withhold the furosemide. C Administer potassium. D Document the findings and reassess in 1 hour.

A Seeing yellow or green halos around lights is a symptom of digoxin toxicity. The nurse should evaluate the patient's digoxin levels.

Which statement indicates the patient understands discharge instructions regarding cholestyramine (Question)? A "I will take Questran 1 hour before my other medications." B "I will increase fiber in my diet." C "I will weigh myself weekly." D "I will have my blood pressure checked weekly."

B Cholestyramine can cause constipation; thus, increasing fiber in the diet is appropriate. All other drugs should be taken 1 hour before or 4 hours after cholestyramine to facilitate proper absorption.

A patient diagnosed with hypercholesterolemia is prescribed lovastatin (Mevacor). Based on this medication order, the nurse will contact the health care provider about which reported condition in the patient's history? A Chronic pulmonary disease B Hepatic disease C Leukemia D Renal disease

B Lovastatin (Mevacor) can cause an increase in liver enzymes and thus should not be used in patients with preexisting liver disease.

A patient is prescribed a noncardioselective beta1 blocker. What nursing intervention is a priority for this patient? A Assessment of blood glucose levels B Respiratory assessment C Orthostatic blood pressure assessment D Teaching about potential tachycardia

B Noncardioselective beta blockers can cause bronchospasms, and a respiratory assessment is indicated to check for potential respiratory side effects. Assessment of blood glucose and teaching about tachycardia will not be priorities

Question 57 A home care nurse is making a routine visit to a client receiving digoxin (Lanoxin) in the treatment of heart failure. The nurse would particularly assess the client for: A Thrombocytopenia and weight gain B Anorexia, nausea, and visual disturbances C Diarrhea and hypotension D Fatigue and muscle twitching

B The first signs and symptoms of digoxin toxicity in adults include abdominal pain, N/V, visual disturbances (blurred, yellow, or green vision, halos around lights), bradycardia, and other dysrhythmias.

Which substance can cause an adverse interaction for a patient taking a loop diuretic? A Spironolactone B Warfarin C Licorice D Gingko

B Loop diuretics are highly protein bound and can displace other protein bound drugs, such as warfarin.

A nurse is giving instruction to a client who is receiving Cholestyramine (Questran) for the treatment of hyperlipidemia. Which of the following statements made by the client indicates the need for further instructions? A "This medication comes in a powder that must be mixed with juice or water before administration". B "I will avoid eating foods rich in saturated fats". C "I will continue taking nicotinic acid as part of the treatment". D "Constipation, belching and heartburn are some of the side effects

C A combination of Cholestyramine (Questran) and nicotinic acid damages the liver. Options A, B, and D are true regarding this medication

The nurse is caring for several patients who are all being treated for hypertension. Which patient will the nurse assess first? A The patient who has been on beta blockers for 1 day B The patient who is on a beta blocker and a thiazide diuretic C The patient who has stopped taking a beta blocker due to cost D The patient who is taking a beta blocker and Lasix (furosemide)

C Abrupt discontinuation of the antihypertensive drug may cause rebound hypertension. The patient who has just been started on an antihypertensive drug and the patients who are on combinations of antihypertensive drugs will not be as high priorities for assessment since they seem to be complying with treatment. Abruptly discontinuing the drug indicates either a failure to understand the treatment or a noncompliance with treatment.

Which foods will the nurse recommend to minimize the effect of potassium losses caused by diuretic medication? A Bread B Cheese C Bananas D Lettuce

C Bananas are excellent sources of potassium.A

A calcium channel blocker has been ordered for a patient. Which condition in the patient's history is a contraindication to this medication? A Hypokalemia B Dysrhythmias C Hypotension D Increased intracranial pressure

C Calcium channel blockers cause vasodilation and thus a drop in blood pressure. They are contraindicated in the presence of hypotension.

A patient with acute pulmonary edema is receiving furosemide (Lasix). What assessment finding indicates to the nurse that the intervention is working? A Potassium level decreased from 4.5 to 3.5 mEq/L B Improvement in mental status C Lungs clear D Output 30 mL/hr

C Furosemide (Lasix) is a potent, rapid-acting diuretic that would be the drug of choice to treat acute pulmonary edema. Furosemide should not cause a drastic change in output or decrease in potassium level, and there is no evidence that it will create any change in mental status.

Before the nurse administers isosorbide mononitrate (Imdur), what is a priority nursing assessment? A Assess serum electrolytes. B Measure blood urea nitrogen and creatinine. C Assess blood pressure. D Monitor level of consciousness.

C Isosorbide mononitrate (Imdur) is a vasodilator and thus can cause hypotension. It is important to assess blood pressure before administering.

After the nurse has finished teaching a patient about the use of sublingual nitroglycerin (Nitrostat), which patient statement indicates that the teaching has been effective? A "I can expect some nausea as a side effect of nitroglycerin." B "I should only take the nitroglycerin if I start to have chest pain." C "I will call an ambulance if I still have pain after taking 3 nitroglycerin 5 minutes apart." D "Nitroglycerin helps prevent a clot from forming and blocking blood flow to my heart."

C The emergency medical services (EMS) system should be activated when chest pain or other symptoms are not completely relieved after 3 sublingual nitroglycerin tablets taken 5 minutes apart.

Question 14 A client with pulmonary edema has been on diuretic therapy. The client has an order for additional furosemide (Lasix) in the amount of 40 mg IV push. Knowing that the client also will be started on Digoxin (Lanoxin), a nurse checks the client's most recent: A Digoxin level B Sodium level C Potassium level D creatine level

C The serum potassium level is measured in the client receiving digoxin and furosemide. Heightened digitalis effect leading to digoxin toxicity can occur in the client with hypokalemia. Hypokalemia also predisposes the client to ventricular dysrhythmias.

The nurse is reviewing a medication history on a patient taking an ACE inhibitor. The nurse plans to contact the health care provider if the patient is also taking which medication? A Docusate sodium (Colace) B Furosemide (Lasix) C Morphine sulfate D Spironolactone (Aldactone)

D ACE inhibitors block the conversion of angiotensin I to angiotensin II, thus also blocking the stimulus for aldosterone production. Aldosterone is responsible for potassium excretion—decreased aldosterone can result in increased serum potassium levels. Spironolactone is a potassium-sparing diuretic and should not be administered with an ACE inhibitor.

Nadolol (Corgard) is prescribed for a patient with chronic stable angina and left ventricular dysfunction. To determine whether the drug is effective, the nurse will monitor for A decreased blood pressure and heart rate. B fewer complaints of having cold hands and feet. C improvement in the strength of the distal pulses. D the ability to do daily activities without chest pain.

D Because the medication is ordered to improve the patient's angina, effectiveness is indicated if the patient is able to accomplish daily activities without chest pain.

Which statement made by the patient demonstrates a need for further instruction regarding the use of nitroglycerin? A "If I get a headache, I should keep taking nitroglycerin and use Tylenol for pain relief." B "I should keep my nitroglycerin in a cool, dry place." C "I should change positions slowly to avoid getting dizzy." D "I can take up to five tablets at 3-minute intervals for chest pain if necessary."

D Patients are taught to take up to three tablets every 5 minutes. If no relief from chest pain is obtained after one tablet, they should seek medical assistance and take up to two more tablets. All other responses demonstrate a good understanding by the patient.

The nurse is monitoring a patient during IV nitroglycerin infusion. Which assessment finding will cause the nurse to take action? A Blood pressure 110/90 mm Hg B Flushing C Headache D Chest pain

D The patient should not continue to have chest pain while on IV nitroglycerin. This would prompt the nurse to intervene. Blood pressure of 110/90 mm Hg is not cause for concern and is expected with nitroglycerin. Headache and flushing are common side effects of nitroglycerin.

The client is admitted with supraventricular tachycardia at a rate of 140 beats per minute. The client's blood pressure is 110/55 mm Hg, and he is asymptomatic except for a "fluttering feeling" in his chest. Which of the following treatments would be appropriate? Select all that apply: a. Intravenous adenosine (Adenocard) b. Intravenous beta blockers c. Intravenous calcium channel blockers d. Intravenous amiodarone e. Emergent cardioversion

a b c d Medications that may be used include adenosine, beta blockers, calcium channel blockers, and amiodarone. If the rate is over 150 beats per minute and the client is symptomatic, emergent cardioversion is considered.

The client has been receiving spironolactone (Aldactone) 50 mg/day for heart failure. The nurse should closely monitor the client for which condition? a. Hypokalemia b. Hyperkalemia c. Hypoglycemia d. Hypermagnesemia

b

For two months a client has been taking several medications to manage chronic heart failure, including furosemide (Lasix) 40 mg by mouth twice a day. The client develops severe muscle cramps and fatigue, and laboratory tests confirm the presence of hypokalemia. Potassium chloride intravenously (IV) and ECG monitoring have been prescribed. Which ECG change associated with hypokalemia should the nurse expect to observe? a. Inverted P waves b. Depressed T waves c. Prolonged PR intervals d. Prolonged QT intervals

b A depressed T wave is associated with hypokalemia. A depressed T wave indicates a problem with ventricular repolarization, a process involved in muscle contraction.

A client who develops heart failure has a serum potassium level of 2.3 mEq/L. Digoxin (Lanoxin) and potassium chloride are prescribed. What action should the nurse take? a. Double the dose of potassium chloride and administer it with the prescribed digoxin. b. Hold the dose of digoxin, administer the potassium chloride, and call the health care provider immediately. c. Give the digoxin and potassium chloride as prescribed and report the laboratory results to the health care provider. d. Administer the prescribed digoxin and potassium chloride with a glass of orange juice and continue to monitor the client.

b A low potassium level with the administration of digoxin can cause digitalis toxicity, resulting in life-threatening dysrhythmias.

A health care provider prescribed 10 mg of morphine immediately and then every 4 hours for a client who had a myocardial infarction. What clinical response will be reduced if the client experiences the intended therapeutic effect of morphine? a. Respiratory rate b. Workload of the heart c. Size of the clot blocking the coronary artery d. Metabolites within the ischemic heart muscle

b Morphine reduces pain and anxiety that limits the response of the sympathetic nervous system, ultimately decreasing cardiac preload and the workload of the heart.

During an admission assessment, the client states that she takes amlodipine (Norvasc). The nurse wishes to determine whether or not the client has any common side effects of a calcium channel blocker. The nurse asks the client if she has which signs and symptoms? (Select all that apply.) a. Insomnia b. Dizziness c. Headache d. Angioedema e. Ankle edema f. Hacking cough

b c e

A client has heart failure and is prescribed Lasix. The nurse is aware that furosemide (Lasix) is what kind of drug? a. Thiazide diuretic b. Osmotic diuretic c. High-ceiling (loop) diuretic d. Potassium-sparing diuretic

c

A client is prescribed ezetimibe (Zetia). Which assessment finding will require immediate action by the nurse? a. Headache. b. Slight nausea. c. Muscle pain. d. Fatigue.

c

A client taking spironolactone (Aldactone) has been taught about the therapy. Which menu selection indicates that the client understands teaching related to this medication? a. Apricots b. Bananas c. Fish d. Strawberries

c

A client's blood pressure (BP) is 145/90. According to the guidelines for determining hypertension, the nurse realizes that the client's BP is at which stage? a. Normal b. Prehypertension c. Stage 1 hypertension d. Stage 2 hypertension

c

A nurse admits a client diagnosed with pneumonia. The client has a history of chronic renal insufficiency, and the health care provider orders furosemide (Lasix) 40 mg twice a day. What is most important to include in the teaching plan for this client? a. That the medication will have to be monitored very carefully owing to the client's diagnosis of pneumonia. b. The fact that Lasix has been proven to decrease symptoms with pneumonia. c. The fact that Lasix has shown efficacy in treating persons with renal insufficiency. d. That the medication will need to be given at a higher than normal dose owing to the client's medical problems.

c

When a patient is receiving diuretic therapy, which of the following would best reflect the patient's fluid volume? A.Blood pressure and pulse B.Serum potassium and sodium levels C.Intake, output, and daily weights D.Measurements of abdominal girth and calf circumference

c

The nurse is monitoring a client during IV nitroglycerin infusion. Which assessment finding will cause the nurse to take action? a. Blood pressure 110/90 mm Hg b. Flushing c. Headache d. Chest pain

d

The nurse is reviewing a medication history on a client taking an ACE inhibitor. The nurse plans to contact the health care provider if the client is also taking which medication? a. docusate sodium (Colace) b. furosemide (Lasix) c. morphine sulfate d. spironolactone (Aldactone)

d

The nurse is scheduled to administer a dose of digoxin (Lanoxin) to an adult client with atrial fibrillation. The client has a potassium level of 4.6 mEq/L. The nurse interprets that the: A. Dose should be omitted only for that day. B. Client needs a dose of potassium before receiving digoxin. C. Dose should be withheld and the healthcare provider notified. D. Dose should be administered as ordered.

d

A patient who is being admitted to the emergency department with intermittent chest pain gives the following list of medications to the nurse. Which medication has the most immediate implications for the patient's care? A Sildenafil (Viagra) B Furosemide (Lasix) C Captopril (Capoten) D Warfarin (Coumadin)

A The nurse will need to avoid giving nitrates to the patient because nitrate administration is contraindicated in patients who are using sildenafil because of the risk of severe hypotension caused by vasodilation.

Which patient assessment would assist the nurse in evaluating therapeutic effects of a calcium channel blocker? A Absence of chest pain B Decreased swelling in the ankles and feet C Patient denies dizziness. D Patient states that she feels stronger.

A The workload in the heart should be decreased with the vasodilation from the calcium channel blocker. With less strain, the patient should have fewer incidences of angina as afterload is decreased.

The nurse knows that the patient has understood the teaching about furosemide when the patient makes which statement? A "I will lower myself slowly into bed." B "I will weigh myself daily on the same scale." C "I will take this medication an hour before eating breakfast." D "I don't need to use sunscreen while I am taking this medication."

B Patients should keep a log of daily weights and report any significant change in weight to their health care provider.

The nurse will suspect that the patient with stable angina is experiencing a side effect of the prescribed metoprolol (Lopressor) if the A patient is restless and agitated. B blood pressure is 90/54 mm Hg. C patient complains about feeling anxious. D cardiac monitor shows a heart rate of 61 beats/minute.

B Patients taking β-adrenergic blockers should be monitored for hypotension and bradycardia. Because this class of medication inhibits the sympathetic nervous system, restlessness, agitation, hypertension, and anxiety will not be side effects.

Question 24 A client with angina complains that the angina pain is prolonged and severe and occurs at the same time each day, most often in the morning, On further assessment a nurse notes that the pain occurs in the absence of precipitating factors. This type of anginal pain is best described as: A Stable angina B Unstable angina C Variant angina D Nonanginal pain

C Stable angina is induced by exercise and is relieved by rest or nitroglycerin tablets. Unstable angina occurs at lower and lower levels of activity and rest, is less predictable, and is often a precursor of myocardial infarction. Variant angina, or Prinzmetal's angina, is prolonged and severe and occurs at the same time each day, most often in the morning.

Question 15 Which of the following classes of medications maximizes cardiac performance in clients with heart failure by increasing ventricular contractility? A Beta-adrenergic blockers B Calcium channel blockers C Diuretics D Inotropic agents

D Inotropic agents are administered to increase the force of the heart's contractions, thereby increasing ventricular contractility and ultimately increasing cardiac output.

A patient with hyperlipidemia has a new order for colesevelam (Welchol). Which nursing action is most appropriate when giving the medication? A Have the patient take this medication with an aspirin. B Administer the medication at the patient's usual bedtime. C Have the patient take the colesevelam with a sip of water. D Give the patient's other medications 2 hours after the colesevelam.

D The bile acid sequestrants interfere with the absorption of many other drugs, and giving other medications at the same time should be avoided.

A patient is prescribed a thiazide diuretic for the treatment of hypertension. When teaching the patient about the medication, which of the following will the healthcare provider include? A "take this medication every day with a large glass of water with your evening meal" B "I'll teach you how to take your radial pulse before taking this medication" C "stop taking this medication if you notice changes in urination" D "eat foods rich in potassium"

D Because thiazide diuretics produce an increase in urine output, the patient should avoid taking the medication in the evening. Potassium is lost in the urine along with sodium and chloride, so the patient should be instructed to include potassium-rich foods in the diet to avoid hypokalemia.

After reviewing the client's chart upon admission to the unit, the nurse consults the health care provider about a new order for lovastatin (Mevacor). What triggered the nurse's action? A Blood glucose of 182 mg/dL B History of peptic ulcers C History of high cholesterol D Elevated liver enzymes

D Treatment with any of the statins for elevated cholesterol and low-density lipoprotein (LDL) levels is contraindicated for clients with active liver disease because these agents can cause increases in liver function.

What instruction should the nurse provide to the client who needs to apply nitroglycerin ointment? a. Use the fingers to spread the ointment evenly over a 3-inch area. b. Apply the ointment to a nonhairy part of the upper torso. c. Massage the ointment into the skin. d. Cover the application paper with ointment before use.

b

What is the best information for the nurse to provide to the client who is receiving spironolactone (Aldactone) and furosemide (Lasix) therapy? a. "Moderate doses of two different diuretics are more effective than a large dose of one." b. "This combination promotes diuresis but decreases the risk of hypokalemia." c. "This combination prevents dehydration and hypovolemia." d. "Using two drugs increases the osmolality of plasma and the glomerular filtration rate."

b

Which assessment finding will alert the nurse to possible toxic effects of amiodarone? a. Heart rate 100 beats per minute b. Crackles in the lungs c. Elevated blood urea nitrogen d. Decreased hemoglobin

b

A client with hypertensive heart disease, who had an acute episode of heart failure, is to be discharged on a regimen of metoprolol (Toprol-XL) and digoxin (Lanoxin). The nurse expects that metoprolol, when administered with digoxin, may: a. Produce headaches b. Precipitate bradycardia c. Increase blood pressure d. Stimulate nodal conduction

b (both have negative chronotropic effects) Metoprolol and digoxin both exert a negative chronotropic effect, resulting in a decreased heart rate. Metoprolol reduces, not produces, headaches. These drugs may cause hypotension. These drugs may depress nodal conduction.

The nurse acknowledges that beta blockers are as effective as antianginals because they do what? a. Increase oxygen to the systemic circulation. b. Maintain heart rate and blood pressure. c. Decrease heart rate and decrease myocardial contractility. d. Decrease heart rate and increase myocardial contractility.

c

The nurse is assessing a client who is taking furosemide (Lasix). The client's potassium level is 3.4 mEq/L, chloride is 90 mmol/L, and sodium is 140 mEq/L. What is the nurse's primary intervention? a. Mix 40 mEq of potassium in 250 mL D5W and infuse rapidly. b. Administer Kayexalate. c. Administer 2 mEq potassium chloride per kilogram per day IV. d. Administer PhosLo, two tablets three times per day.

c

The nurse is aware that which group(s) of antihypertensive drugs are less effective in African-American clients? a. Diuretics b. Calcium channel blockers and vasodilators c. Beta blockers and ACE inhibitors d. Alpha blockers

c

A client with heart failure has Lanoxin (digoxin) ordered. What would the nurse expect to find when evaluating for the therapeutic effectiveness of this drug? A. diaphoresis with decreased urinary output B. increased heart rate with increase respirations C. improved respiratory status and increased urinary output D. decreased chest pain and decreased blood pressure

c Digoxin, a cardiac glycoside, is used in clients with heart failure to slow and strengthen the heartbeat. As cardiac output is improved, renal perfusion is improved and urinary output increases.

While providing home care to a client with congestive heart failure, the nurse is asked how long diuretics must be taken. What is the nurse's best response? A "As you urinate more, you will need less medication to control fluid." B "You will have to take this medication for about a year." C "The medication must be continued so the fluid problem is controlled." D "Please talk to your health care provider about medications and treatments."

c This is the most therapeutic response and gives the client accurate information.

What statement is the most important for the nurse to include in the teaching plan for a client who has started on a transdermal nitroglycerin patch? a. "This medication works faster than sublingual nitroglycerin works." b. "This medication is the strongest of any nitroglycerin preparation available." c. "This medication should be used only when you are experiencing chest pain." d. "This medication will work for 24 hours and you will need to change the patch daily

d

The nurse is reviewing the medication that have been ordered for a patient for whom a loop diuretic has been newly prescribed. The loop diuretic may have a possible interaction with which of the following? A.Vitamin C B.Warfarin C.Penicillins D. NSAID's

d When loop diuretics and NSAID's are taken concurrently, they can decrease the diuretic effect because these two drug classes have opposite effects on renal prostaglandin activity. Loop diuretics activate renal prostaglandins which result in dilation of blood vessels reducing renal, pulmonary, and systemic vascular resistance. NSAIDS inhibit prostaglandin activity.

The nurse would question an order for colesevelam (Welchol) if the patient has which condition in the medical history? A Impaction B Glaucoma C Hepatic disease D Renal disease

A Colesevelam (Welchol) binds with bile in the intestinal tract to form an insoluble complex. It can also bind to other substances and lead to intestinal obstruction.

Which client will the nurse assess first? a. The client who has been on beta blockers for 1 day. b. The client who is on a beta blocker and a thiazide diuretic. c. The client who has stopped taking a beta blocker due to cost. d. The client who is taking a beta blocker and Lasix (furosemide).

c

Diltiazem (Cardizem) is ordered for a patient with newly diagnosed Prinzmetal's (variant) angina. When teaching the patient, the nurse will include the information that diltiazem will A reduce heart palpitations. B decrease spasm of the coronary arteries. C increase the force of the heart contractions. D help prevent plaque from forming in the coronary arteries.

B Prinzmetal's angina is caused by coronary artery spasm. Calcium channel blockers (e.g., diltiazem, amlodipine [Norvasc]) are a first-line therapy for this type of angina.

A patient is receiving intravenous furosemide. What medical condition might the nurse anticipate the patient reporting during an admission health history? A Acute heart failure B Liver cirrhosis C Peripheral edema D Primary hypertension

A IV furosemide is usually indicated in emergency conditions requiring diuresis and control of blood pressure.

Question 3 Toxicity from which of the following medications may cause a client to see a green-yellow halo around lights? A Digoxin B Furosemide (Lasix) C Metoprolol (Lopressor) D Enalapril (Vasotec)

A One of the most common signs of digoxin toxicity is the visual disturbance known as the "green-yellow halo sign." The other medications aren't associated with such an effect.

Which of the following medications should be held if the pulse is less than 50 bpm? A Digoxin B Atenolol C Metoprolol D Propranolol E Diltiazem

B C D Digoxin should be held is pulse is less than 60 bpm, diltiazem is not contraindicated for bpm

Nesiritide (Natrecor), a vasodilator, is prescribed for a client with acute heart failure and pulmonary edema. The nurse is assessing the client's response to the medication. Which clinical manifestation should decrease when the medication is effective? a. Dyspnea b. Hypotension c. Unstable angina d. Premature heartbeats

a Human B-type natriuretic peptide binds to receptors in vascular smooth muscle and endothelial cells, leading to smooth muscle relaxation. Dyspnea will decrease as a result of the action of nesiritide.

The nurse is monitoring a client taking digoxin (Lanoxin) for treatment of heart failure. Which assessment finding indicates a therapeutic effect of the drug? a. Heart rate 110 beats per minute b. Heart rate 58 beats per minute c. Urinary output 40 mL/hr d. Blood pressure 90/50 mm Hg

b

Which client would the nurse need to assess first if the client is receiving mannitol (Osmitrol)? a. A 67-year-old client with type 1 diabetes mellitus b. A 21-year-old client with a head injury c. A 47-year-old client with anuria d. A 55-year-old client receiving cisplatin to treat ovarian cancer

c

The client diagnosed with coronary artery disease is prescribed atorvastatin (Lipitor), an HMG-CoA reductase inhibitor. Which statement by the client warrants the nurse notifying the health-care provider? 1. "I really haven't changed my diet, but I am taking my medication every day." 2. "I am feeling pretty good except I am having muscle pain all over my body." 3. "I am swimming at the local pool about three times a week for 30 minutes." 4. "I am taking this medication first thing in the morning with a bowl of oatmeal."

2 Statins can cause muscle injury, which can lead to myosititis, fatal rhabdomy- olysis, or myopathy. Muscle pain or tenderness should be reported to the HCP immediately; usually the medica- tion is discontinued.

The client with a serum cholesterol level of 320 mg/dL is taking the antihyperlipidemic medication ezetimibe (Zetia). Which statement by the client indicates the client needs more teaching concerning this medication? 1. "This medication helps decrease the absorption of cholesterol in my intestines." 2. "I cannot take this medication with any other cholesterol-lowering medication." 3. "I need to eat a low-fat, low-cholesterol diet even when taking the medication." 4. "It will take a few months for my cholesterol level to get down to normal levels."

2 This is not a true statement; therefore, the client needs more teaching. Zetia acts by decreasing cholesterol absorp- tion in the intestine and is used together with statins to help lower cholesterol in clients whose cholesterol levels cannot be controlled by taking statins alone.

Hydrochlorothiazide has been prescribed for a patient who is pre-diabetic. The nurse knows that patient teaching has been successful when the patient makes which statement? A "I will have my blood sugar checked periodically." B "I can take herbal supplements with this medication." C "I will stop eating fresh fruit because it has too much sugar." D "I don't need to check my blood sugar while taking this medication."

A Because large doses of hydrochlorothiazide increase serum glucose levels, a patients who are pre-diabetic should have their blood pressure checked regularly.

Which laboratory value will the nurse report to the health care provider as a potential adverse response to hydrochlorothiazide? A Sodium level of 140 mEq/L B Fasting blood glucose level of 140 mg/dL C Calcium level of 9 mg/dL D Chloride level of 100 mEq/L

B Hydrochlorothiazide can cause hyperglycemia. Normal calcium level is approximately 8.8-10.3 mg/dL; normal sodium level is 135-147 mEq/L; normal chloride level is 95-107 mEq/L, and normal fasting blood glucose should be 60-110 mg/dL.

The patient taking methyldopa (Aldomet) has elevated liver function tests. What is the nurse's best action? A Document the finding and continue care. B Notify the health care provider. C Immediately stop the medication. D Change the patient's diet.

B This drug should not be used in patients with impaired liver function. The nurse should notify the health care provider so that the patient can be tapered off the medication. The nurse should not immediately stop this medication as the patient could have a hypertensive crisis. The patient's diet is not the cause of elevated liver enzymes and should not make a difference with therapy.

A patient has been on a statin drug for about six weeks and wants to stop taking it because he is on "too many pills." What does the nurse tell the patient may occur if he abruptly discontinues the statin? A LDL level return to pretreatment level B Leg pain related to myopathy C Death from acute myocardial infarction D Rebound increase in serum liver enzymes

C Abruptly discontinuing the use of statin drugs does not cause a rebound increase in serum liver enzymes.

A nurse is caring for a patient receiving acetazolamide (Diamox). Which assessment finding will require immediate nursing intervention? A A decrease in bicarbonate level B An increase in urinary output C A decrease in arterial pH D An increase in PaO2

C Acetazolamide (Diamox) causes excretion of bicarbonate, which would worsen metabolic acidosis. It is used to treat metabolic alkalosis, edema, seizures, and acute glaucoma. A decrease in blood pH would indicate that the patient was becoming more acidotic.

Which statement made by the patient indicates to the nurse that understanding about discharge instructions on antihyperlipidemic medications has occurred? A "Antihyperlipidemic medications will replace the other interventions I have been doing to try to decrease my cholesterol." B "It is important to double my dose if I miss one in order to maintain therapeutic blood levels." C "I will stop taking the medication if it causes nausea and vomiting." D "I will continue my exercise program to help increase my high-density lipoprotein serum levels."

D Antihyperlipidemic medications are an addition to, not a replacement for, the therapeutic regimen used to decrease serum cholesterol levels. The dose should never be doubled if one is missed nor stopped due to side effects.

A client is taking furosemide (Lasix) and digoxin (Lanoxin) for heart failure. Why does the nurse advise the client to drink a glass of orange juice every day? a. Maintaining potassium levels b. Preventing increased sodium levels c. Correcting the associated dehydration d. Limiting the drugs' synergistic effects

a Orange juice is an excellent source of potassium. Furosemide promotes excretion of potassium, which can result in hypokalemia.

A client diagnosed with cirrhosis of the liver and ascites is receiving Spironolactone (Aldactone). The nurse understands that this medication spares elimination of which element? A Sodium B Potassium C Phosphate D Albumin

b If ascites is present in the client with cirrhosis of the liver, potassium-sparing diuretics such as Aldactone should be administered because it inhibits the action of aldosterone on the kidneys.

When reviewing the electrolyte panel results of a patient taking furosemide for treatment of hypertension, the nurse observes that the patient's potassium level is 3.2 mEq/L. Which diuretic will the nurse administer to the patient to counteract this effect of furosemide? A Furosemide B Spironolactone C Bumetanide D Hydrochlorothiazide

b Spironolactone is a potassium-sparing diuretic primarily used to counteract potassium loss in patients taking loop or thiazide diuretics.

Which information given by a patient admitted with chronic stable angina will help the nurse confirm this diagnosis? a. The patient states that the pain "wakes me up at night." b. The patient rates the pain at a level 3 to 5 (0 to 10 scale). c. The patient states that the pain has increased in frequency over the last week. d. The patient states that the pain "goes away" with one sublingual nitroglycerin tablet.

d Chronic stable angina is typically relieved by rest or nitroglycerin administration.

Which statement indicates to the nurse that the client with coronary artery disease (CAD) understands the medication teaching for taking aspirin, an antiplatelet, daily? 1. "I will probably have occasional bleeding when taking this medication." 2. "I will call 911 if I have chest pain unrelieved and I will chew an aspirin." 3. "If I have any ringing in my ears, I will call my health-care provider." 4. "I should take my daily aspirin on an empty stomach for better absorption."

2 Aspirin is administered as an antiplatelet to prevent coronary artery occlusion. It is not administered for chest pain. If the client has chest pain that is not relieved with NTG, the client should call the Emergency Medical Services (EMS) and get medical treatment immediately. Taking an extra aspirin may prevent further cardiac damage.

The nurse is preparing to administer the alpha-beta blocker labetalol (Normodyne) intravenous push (IVP) to a client diagnosed with hypertensive crisis. Which interven- tion should the nurse implement? 1. Monitor the client's labetalol serum drug level. 2. Keep the medication covered with tin foil. 3. Administer the medication slow IVP over 5 minutes. 4. Teach the client signs/symptoms of hypertension.

3 Medications that directly affect the cardiac muscle or vasculature are administered slowly over a minimum of 5 minutes for safety reasons. Many medications require dilution with normal saline to have suffi- cient volume for a smooth equal delivery to prevent cardiac dysrhythmias.

According to the American Heart Association (AHA), which medication should the client suspected of having a myocardial infarction take immediately when having chest pain? 1. Morphine, a narcotic analgesic. 2. Acetaminophen (Tylenol), a nonnarcotic analgesic. 3. Acetylsalicylic acid (aspirin), an antiplatelet. 4. Nitroglycerin paste, a coronary vasodilator.

3 The AHA recommends that a client hav- ing chest pain chew two baby aspirins or one 325-mg tablet immediately to help prevent platelet aggregation and further extension of a coronary thrombosis.

The nurse is preparing to administer nitroglycerin, a coronary vasodilator transdermal patch, to the client diagnosed with a myocardial infarction. Which intervention should the nurse implement? 1. Question applying the patch if the client's BP is less than 110/70. 2. Use nonsterile gloves when applying the transdermal patch. 3. Date and time the transdermal patch prior to applying to client's skin. 4. Place the transdermal patch on the site where the old patch was removed.

3 The nurse should remove the old patch, wash the client's skin, note the date and time the new patch is applied, and apply it in a new area that is not

The client newly diagnosed with coronary artery disease is being prescribed a daily aspirin. The client tells the nurse, "I had a bad case of gastritis last year." Which intervention should the nurse implement first? 1. Ask the client if he or she informed the HCP of the gastritis. 2. Explain that regular aspirin could cause gastric upset. 3. Instruct the client to take an enteric-coated aspirin. 4. Determine if the client is taking any antiulcer medication.

4 Assessment is the first part of the nurs- ing process, and determining if the client is taking any antiulcer medication is the first question the nurse should ask the client.

12. The nurse is preparing to administer digoxin (Lanoxin), a cardiac glycoside, to a client diagnosed with congestive heart failure. Which area should the nurse assess prior to administering the medication? 1. A 2. B 3. C 4. D

4 The apical pulse located at the fifth intercostal midclavicular space must be assessed for 1 minute prior to adminis- tering digoxin. If the apical pulse is less than 60 beats per minute, the nurse should hold the medication.

The nurse is completing A.M. care with a client diagnosed with angina when the client complains of chest pain. The client has a saline lock in the right forearm. Which intervention should the nurse at the bedside implement first? 1. Assess the client's vital signs. 2. Administer sublingual nitroglycerin (NTG). 3. Administer intravenous morphine sulfate (MS) via saline lock. 4. Administer oxygen via nasal cannula.

4 The nurse would have oxygen at the bedside, and applying it would be the first intervention the nurse could implement at the bedside.

Which client should the nurse most likely suspect will require polypharmacy to control essential hypertension? 1. The 84-year-old white male client. 2. The 22-year-old Hispanic female client. 3. The 60-year-old Asian female client. 4. The 46-year-old African American male client.

4 Ethnically and racially, African Americans have poorer responses to ACE inhibitors, beta blockers, and other antihypertensive medications than do people of other backgrounds. There is no specific reason known for this, but it is empirically and scientifically documented. Polyphar- macy is using multiple medications to medically treat a client, and African Americans often require this to treat hypertension.

The nurse is preparing to administer medications to the following clients. Which client should the nurse question administering the medication? 1. The client receiving the angiotensin-receptor blocker losartan (Cozaar) who has a BP of 168/94. 2. The client receiving the calcium channel blocker diltiazem (Cardizem) who has 1+ nonpitting edema. 3. The client receiving the alpha blocker terazosin (Hytrin) who is complaining of a headache. 4. The client receiving the thiazide diuretic hydrochlorothiazide (HCTZ) who is complaining of leg cramps.

4 Leg cramps could indicate hypokalemia, which may lead to life-threatening cardiac dysrhythmias. Therefore, the nurse should question administering this medication until a serum potassium level is obtained.

The client diagnosed with congestive heart failure is taking digoxin (Lanoxin), a cardiac glycoside. Which data indicates the medication is effective? 1. The client's blood pressure is 110/68. 2. The client's apical pulse rate is regular. 3. The client's potassium level is 4.2 mEq/L. 4. The client's lungs are clear bilaterally.

4 Signs and symptoms of CHF are crack- les in the lungs, jugular vein distention, and pitting edema. Therefore, if the client has clear lung sounds, the nurse can assume the medication is effective.

A client is receiving digoxin (Lanoxin) 0.25 mg. Daily. The health care provider has written a new order to give metoprolol (Lopressor) 25 mg. B.I.D. In assessing the client prior to administering the medications, which of the following should the nurse report immediately to the health care provider? A Blood pressure 94/60 B Heart rate 76 C Urine output 50 ml/hour D Respiratory rate 16

A Both medications decrease the heart rate. Metoprolol affects blood pressure. Therefore, the heart rate and blood pressure must be within normal range (HR 60-100; systolic B/P over 100) in order to safely administer both medications.

Which assessment finding will alert the nurse to suspect early digitalis toxicity? A Loss of appetite with slight bradycardia Correct B Blood pressure of 90/60 mm Hg C Heart rate of 110 beats per minute D Confusion and diarrhea

A Early symptoms of digitalis toxicity include anorexia, nausea and vomiting, and bradycardia.

Which assessment indicates to the nurse a therapeutic effect of mannitol (Osmitrol) has been achieved? A Decreased intracranial pressure B Decreased potassium C Increased urine osmolality D Decreased serum osmolality

A Mannitol (Osmitrol) is an osmotic diuretic that pulls fluid from extravascular spaces into the bloodstream to be excreted in urine. This will decrease intracranial pressure, increase excretion of medications, decrease urine osmolality, and increase serum osmolality.

A client with hypercholesterolemia and atherosclerosis is prescribed nicotinic acid (Niaspan). Which instruction does the nurse provide the client? A "This medication may make you flush." B "Take this medication on an empty stomach." C "You will not need to change your diet with this medication." D "Take this medication when you experience chest pain."

A Nicotinic acid causes increased release of prostaglandins, resulting in vasodilation. Clients may experience flushing and a very warm feeling all over. Taking the drug with meals minimizes this side effect.

Question 19 A nurse is preparing for the admission of a client with heart failure who is being sent directly to the hospital from the physician's office. The nurse would plan on having which of the following medications readily available for use? A Diltiazem (Cardizem) B Digoxin (Lanoxin) C Propranolol (Inderal) D Metoprolol (Lopressor)

B Digoxin exerts a positive inotropic effect on the heart while slowing the overall rate through a variety of mechanisms. Digoxin is the medication of choice to treat heart failure. Diltiazem (calcium channel blocker) and propranolol and metoprolol (beta blockers) have a negative inotropic effect and would worsen the failing heart.

Which assessment finding in a patient taking an HMG-CoA reductase inhibitor will the nurse act on immediately? A Decreased hemoglobin B Elevated liver function tests C Elevated HDL D Elevated LDL

B HMG-CoA reductase inhibitors (statins) can cause hepatic toxicity; thus, it is necessary to monitor liver function tests. The nurse should act on this finding immediately. Decreased hemoglobin should be addressed but not immediately. It is most likely not related to the administration of the HMG-CoA reductase inhibitor. Also, while an elevated LDL level must be addressed, it is not as high a priority as the elevated liver function test results. An elevated HDL is a positive finding and an encouraging result.

Why do loop diuretics have a greater diuresis effect than thiazides or potassium-sparing diuretics? A Loop diuretics act directly by blocking sodium-potassium exchange in the distal convoluted tubule. B Loop diuretics act early in the loop of Henle and block sodium chloride reabsorption. C Loop diuretics block greater amounts of sodium chloride reabsorption in the proximal convoluted tubule. D Loop diuretics promote the greatest reabsorption of sodium chloride in the loop of Henle.

B Loop diuretics block the reabsorption of larger volumes of sodium and water early in the loop of Henle to promote diuresis.

Question 58 A 55-year-old client is admitted with an acute inferior-wall myocardial infarction. During the admission interview, he says he stopped taking his metoprolol (Lopressor) 5 days ago because he was feeling better. Which of the following nursing diagnoses takes priority for this client? A Anxiety B Ineffective tissue perfusion; cardiopulmonary C Acute pain D Ineffective therapeutic regimen management

B MI results from prolonged myocardial ischemia caused by reduced blood flow through the coronary arteries. Therefore, the priority nursing diagnosis for this client is Ineffective tissue perfusion (cardiopulmonary). Anxiety, acute pain, and ineffective therapeutic regimen management are appropriate but don't take priority.

A client is admitted to the hospital with a diagnosis of heart failure and acute pulmonary edema. The health care provider prescribes furosemide (Lasix) 40 mg intravenous (IV) stat to be repeated in one hour. To best evaluate the effectiveness of the furosemide, the nurse should: A Perform daily weights. B Auscultate breath sounds. C Monitor intake and output. D Assess for dependent edema.

B Maintaining adequate gas exchange and minimizing hypoxia with pulmonary edema are critical; therefore, assessing the effectiveness of furosemide therapy as it relates to the respiratory system is most important. Furosemide inhibits the reabsorption of sodium and chloride from the loop of Henle and distal renal tubule causing diuresis; as diuresis occurs fluid moves out of the vascular compartment, thereby reducing pulmonary edema and the bilateral crackles.

What is the best information for the nurse to provide to the patient who is receiving spironolactone (Aldactone) and furosemide (Lasix) therapy? A "Moderate doses of two different diuretics are more effective than a large dose of one." B "This combination promotes diuresis but decreases the risk of hypokalemia." C "This combination prevents dehydration and hypovolemia." D "Using two drugs increases the osmolality of plasma and the glomerular filtration rate."

B Spironolactone is a potassium-sparing diuretic; furosemide causes potassium loss. Giving these together minimizes electrolyte imbalance. It is not accurate to state that the drug combination prevents dehydration and hypovolemia or that it increases the osmolality of plasma and the glomerular filtration rate. Stating that giving two different diuretics is more effective is not specific enough information for the patient.

A nurse is caring a client who is taking digoxin (Lanoxin) 0.25mcg tab once a day. The client suddenly complaints of anorexia, nausea, vomiting, and diarrhea. The physician is ruling a digoxin toxicity. As a nurse, you know the therapeutic digoxin rate is? A 0.25-0.5 ng/ml. B 0.5-2 ng/ml. C 1.5-3 ng/ml. D 3.5-4.5 ng/ml.

B The therapeutic level of digoxin is 0.5-2 ng/ml.

Why do patients taking thiazide diuretics have an increased risk for hypercalcemia? A They promote GI absorption of calcium. B They promote calcium reabsorption. C They decrease the metabolism of calcium. D They cause calcium loss in the urine.

B Thiazide diuretics influence the mechanism of sodium, chloride, and calcium transport to favor calcium reabsorption.

Which of the following co-morbidities increase a patient's risk in developing digoxin toxicity? A. Heart Failure B. Renal Failure C. Parkinson's disease D. Dementia

B Because 60-90% of digoxin is excreted unchanged by the kidneys, even the modest renal impairment can dramatically hasten the accumulation of digoxin to toxic concentrations in the body.

A patient who began taking furosemide 24 hours ago complains of increased frequency of urination. How should the nurse respond to the patient? A "Increased frequency of urination is a side effect of this medication, and there is not much we can do about it." B "Frequency of urination will decrease 6-8 hours after you take this medication. You should take it in the morning." C "It is okay to skip a dose of your medication occasionally if frequent urination is bothering you." D "I can provide a bedpan or urinal to prevent frequent visits to the bathroom."

B Scheduling diuretic medication during the day time promotes sleep at night by preventing nocturia.

A nurse is interviewing a client who is about to receive metoprolol. Upon the history taking, the client is also taking insulin. Which of the following statements made by the nurse will correctly explain the possible interaction of these medications? A "This medication will maintain the blood sugar level on a normal range". B "This medication will have no effect in the blood sugar level". C "This medication may mask some of the symptoms of hypoglycemia such as tremor, palpitation, and rapid heartbeat. D "This medication may mask some of the symptoms of hyperglycemia such as headache, increased thirst, and blurred vision".

C Beta-blockers such as metoprolol may increase the risk of hypoglycemia in patients receiving insulin. In addition, beta-blockers may mask some of the symptoms of hypoglycemia such as tremor, palpitation, and rapid heartbeat, making it more difficult to recognize an oncoming episode.

A patient is being treated for short-term management of heart failure with milrinone (Primacor). What is the primary nursing action? A Administer digoxin via IV infusion with the milrinone. B Administer furosemide (Lasix) via IV infusion after the milrinone. C Monitor blood pressure continuously. D Maintain an infusion of lactated Ringer's with milrinone infusion.

C Milrinone lactate (Primacor) is a phosphodiesterase inhibitor administered intravenously for short-term treatment in patients with heart failure not responding adequately to digoxin, diuretics, or other vasodilators. Blood pressure and heart rate should be closely monitored. Digoxin is not administered with the milrinone but is usually tried before treatment with milrinone. Furosemide is not necessarily administered after the milrinone, although it could be. It is not, however, administered routinely via IV infusion. Lactated Ringer's does not have to be administered with milrinone.

During assessment of a patient diagnosed with pheochromocytoma, the nurse auscultates a blood pressure of 210/110 mm Hg. What is the nurse's best action? A Ask the patient to lie down and rest. B Assess the patient's dietary intake of sodium and fluid. C Administer phentolamine (Regitine). D Administer nitroprusside (Nipride).

C Phentolamine (Regitine) is a potent alpha-blocking agent specifically effective for treatment of hypertension associated with pheochromocytoma. The patient's blood pressure is elevated owing to tumor secretion. If the patient lies down, the blood pressure will not necessarily decrease. Increased fluid and sodium is not the cause of hypertension in this condition. Nipride is not the recommended treatment for this condition.

A patient taking prazosin (Minipress) has a blood pressure of 140/90 mm Hg and is complaining of swollen feet. What is the nurse's best action? A Hold the medication. B Call the health care provider. C Determine the patient's history. D Weigh the patient.

C The desired therapeutic effect of prazosin (Minipress) may not fully occur for 4 weeks. The nurse does not know how long the patient has been on this medication. There is no need to hold the medication. It is more important to determine the patient's history prior to weighing the patient or calling the health care provider, since symptoms may be the result of the medication not yet achieving the full therapeutic effect.

A patient is prescribed chlorthalidone (Thalitone). What is the most important information the nurse will teach the patient? A "Do not drink more than 10 ounces of fluid a day while on this medication." B "Take this medication on an empty stomach." C "Take this medication before bed each night." D "Wear protective clothing and sunscreen while taking this medication."

D Adverse effects associated with chlorthalidone include photosensitivity. The nurse should teach the patient to protect himself when out in the sun. There is no evidence that fluid should be restricted while taking the medication, that it should be taken on an empty stomach, or that it should only be taken at bedtime.

Which patient's statement indicates a need for further medication instruction about colestipol (Cholestid)? A "The medication may cause constipation, so I will increase fluid and fiber in my diet." B "I should take this medication 1 hour after or 4 hours before my other medications." C "I might need to take fat-soluble vitamins to supplement my diet." D "I should stir the powder in as small an amount of fluid as possible to maintain potency of the medication."

D Colestipol (Colestid) is a powder that must be well-diluted in fluids before administration to avoid esophageal irritation or obstruction and intestinal obstruction. The powder should not be stirred because it may clump; it should be left to dissolve slowly for at least 1 minute.

The nurse is teaching a patient about side effects of spironolactone. Which statement by the patient indicates teaching has been effective? A "I will use topical medication if I get a rash." B "I will continue using a salt substitute on my eggs every morning." C "I will stop taking this medication if I experience side effects." D "I will do my gardening in the evening after the sun has gone down."

D Photosensitivity is a side effect of potassium-sparing diuretics, including spironolactone. The patient should avoid directly sunlight when taking spironolacto

A patient is started on digoxin therapy. The nurse will teach the patient to avoid taking which of the following food items concurrently with their daily dose? a. High fiber cereals B. Canned fruits C. Spicy foods D. Cooked vegetables

a Do not administer oral preparations of digoxin with high fiber foods. Studies have shown that digoxin binds with fiber, thereby reducing the amount of medication available for absorption.

A client who has angina is prescribed nitroglycerin. The nurse reviews which appropriate nursing interventions for nitroglycerin (Select all that apply.) a. Have the client lie down when taking a nitroglycerin sublingual tablet. b. Teach client to repeat taking a tablet in 5 minutes if chest pain persists. c. Apply Transderm-Nitro patch to a hairy area to protect skin from burning. d. Call the health care provider after taking 5 tablets if chest pain persists. e. Warn client against ingesting alcohol while taking nitroglycerin.

a b e

A client with left ventricular heart failure is taking digoxin (Lanoxin) 0.25 mg daily. What changes does the nurse expect to find if this medication is therapeutically effective? Select all that apply. a. Diuresis b. Tachycardia c. Decreased edema d. decreased pulse rate e. Reduced heart murmur f. Jugular vein distention

a c d Digoxin increases kidney perfusion, which results in urine formation and diuresis. Because of digoxin's inotropic and chronotropic effects, the heart rate will decrease. The urine output increases because of improved cardiac output and kidney perfusion, resulting in a reduction in edema. Digoxin increases the force of contractions (inotropic effect) and decreases the heart rate (chronotropic effect).

A client is prescribed losartan (Cozaar). The nurse teaches the client that an angiotensin II receptor blocker (ARB) acts by doing what? a. Inhibiting angiotensin-converting enzyme b. Blocking angiotensin II from AT1 receptors c. Preventing the release of angiotensin I d. Promoting the release of aldosterone

b

A client's high-density lipoprotein (HDL) is 60 mg/dL. What does the nurse acknowledge concerning this level? a. It is lower than the desired level of HDL. b. It is the desired level of HDL. c. It is higher than the desired level of HDL. d. It is a much lower HDL level than desired.

b

The health care provider is planning to discontinue a client's beta blocker. What instruction should the nurse give the client regarding the beta blocker? a. The beta blocker should be abruptly stopped when another cardiac drug is prescribed. b. The beta blocker should NOT be abruptly stopped; the dose should be tapered down. c. The beta blocker dose should be maintained while taking another antianginal drug. d. Half the beta blocker dose should be taken for the next several weeks.

b

The nurse reviews a client's laboratory values and finds a digoxin level of 10 ng/mL and a serum potassium level of 5.9 mEq/L. What is the nurse's primary intervention? a. To administer atropine b. To administer digoxin immune FAB c. To administer epinephrine d. To administer Kayexalate

b

Which laboratory value will the nurse report to the health care provider as a potential adverse response to hydrochlorothiazide (HydroDIURIL)? a. Sodium level of 140 mEq/L b. Fasting blood glucose level of 140 mg/dL c. Calcium level of 9 mg/dL d. Chloride level of 100 mEq/L

b

Which statement indicates the client understands discharge instructions regarding cholestyramine (Questran)? a. "I will take Questran 1 hour before my other medications." b. "I will increase fiber in my diet." c. "I will weigh myself weekly." d. "I will have my blood pressure checked weekly."

b

The nurse provides medication discharge instructions to a client who received a prescription for digoxin (Lanoxin) following the client's myocardial infarction. The nurse concludes that the teaching was effective when the client says, "I should: a. Avoid foods high in potassium." b. Check my radial pulse rate daily." c. Increase my intake of vitamin K." d. Adjust the dosage according to my activities."

b Checking the radial pulse rate daily is necessary for monitoring cardiac function; digoxin slows and strengthens the heart rate. Digoxin should be withheld and the health care provider notified if the pulse rate falls below a predetermined rate (e.g., 60 beats per minute).

A client with supraventricular tachycardia (SVT) is being treated with diltiazem hydrochloride (Cardizem). What assessment indicates to the nurse that the diltiazem hydrochloride is effective? a. Blood pressure of 90/60 mm Hg b. Heart rate of 110 beats per minute c. No longer complaining of heart palpations d. Increased urine output

b Diltiazem hydrochloride's purpose is to slow the heart rate down. SVT has a heart rate of 150 to 250 beats per minute. A heart rate of 110 indicates that the diltiazem hydrochloride is having the desired effect.

A pregnant woman develops chest pain and is found to be in atrial fibrillation. Which medication would be appropriate to prescribe for this client? a. Warfarin (Coumadin) b. Heparin c. Aspirin d. Atenolol

b Heparin is the only one of the medications that would be used for this problem and does not cross the placental barrier. Warfarin, atenolol, and aspirin cross the placenta and are distributed in breast milk, leading to potential fetal death, neonatal hemorrhage, or intrauterine death.

A client is admitted to the hospital with a diagnosis of heart failure and acute pulmonary edema. The health care provider prescribes furosemide (Lasix) 40 mg intravenous (IV) stat to be repeated in one hour. To best evaluate the effectiveness of the furosemide, the nurse should: a. Perform daily weights. b. Auscultate breath sounds. c. Monitor intake and output. d. Assess for dependent edema.

b Maintaining adequate gas exchange and minimizing hypoxia with pulmonary edema are critical; therefore, assessing the effectiveness of furosemide therapy as it relates to the respiratory system is most important.

While assisting a patient in getting out of bed, what objective indicator (other than a manometer) can the nurse use to detect early postural hypotension? A.Pupil response B.Pulse rate C.State of orientation D.Increased muscle rigidity

b The cardiovascular system first tries to compensate for a sudden drop in circulating volume often as a result of diuretic therapy, by increasing the heart rate by sympathetic stimulation.

A client with a history of cirrhosis of the liver develops heart failure and is experiencing bigeminal premature ventricular complexes. What should the nurse expect about the dose of lidocaine (Xylocaine) prescribed by the health care provider? a. Higher to compensate for the impaired liver function b. Lower because the drug is metabolized at a diminished rate c. Reduced because other organs will compensate for the sluggish liver d. Equal to that needed for other clients to provide a loading dose for the myocardium

b The client has heart failure, which causes liver congestion, further compromising liver function; therefore, less than the usual adult dose will be prescribed because the liver will not be able to break down lidocaine as effectively as necessary. A dose higher to compensate for the impaired liver function increases the concentration of lidocaine in the blood, leading to toxicity. Lidocaine is metabolized by the liver; other organs cannot assist in the process. This may be life threatening because the client cannot metabolize lidocaine at the required rate, and toxicity may result.

A client is ordered furosemide (Lasix) to be given via intravenous push. What interventions should the nurse perform? (Select all that apply.) a. Administer at a rate no faster than 20 mg/min. b. Assess lung sounds before and after administration. c. Assess blood pressure before and after administration. d. Maintain accurate intake and output record. e. Monitor ECG continuously. f. Insert an arterial line for continuous blood pressure monitoring.

b c d

The nurse reviews the history for a client taking atorvastatin (Lipitor). What will the nurse act on immediately? a. Client takes medications with grape juice. b. Client takes herbal therapy including kava kava. c. Client is on oral contraceptives. d. Client was started on penicillin for a respiratory infection.

c

What will the nurse instruct the client to do to prevent the development of tolerance to nitroglycerin? a. Apply the nitroglycerin patch every other day. b. Switch to sublingual nitroglycerin when the client's systolic blood pressure elevates to more than 140 mm Hg. c. Apply the nitroglycerin patch for 14 hours and remove it for 10 hours at night. d. Use the nitroglycerin patch for acute episodes of angina only.

c

A client is receiving furosemide (Lasix) to help treat congestive heart failure. Which of the laboratory results warrants notifying the health care provider? a. Hemoglobin 14.1g/dL b. Hematocrit 46% c. Potassium 3.0mEq/L d. White blood cell count 9200 cells/mm

c A potassium level of 3.0 mEq/L is indicative of hypokalemia. Normal values for an adult are 3.5 to 5.5 mEq/L.

Which of the following is the priority intervention for the nurse prior to administering digoxin (lanoxin)? A. Palpate pedal pulses for quality and strength B. Monitor renal function lab values C. Auscultate the apical pulse for one full minute D. Assess serum potassium levels

c It is critical to auscultate the apical pulse for one full minute prior to administration of digoxin. This is a PRIORITY nursing action. Digoxin slows the heart rate thus the dose would be held for a AP less than 60 bpm. Bradycardia is also a potential sign of digoxin toxicity again warranting the drug be held for AP < 60. Although monitoring renal function and electrolytes are appropriate interventions, they are not the priority.

Which statement indicates to the nurse that the client understands sublingual nitroglycerin medication instructions? a. "I will take up to five doses every 3 minutes for chest pain." b. "I can chew the tablet for the quickest effect." c. "I will keep the tablets locked in a safe place until I need them." d. "I should sit or lie down after I take a nitroglycerin tablet to prevent dizziness."

d

A health care provider prescribes milrinone (Primacor) for a client with a diagnosis of congestive heart failure who was unresponsive to conventional drug therapy. What is most important for the nurse to do first? a. Administer the loading dose over 10 minutes. b. Monitor the ECG continuously for dysrhythmias during infusion. c. Assess the heart rate and blood pressure continuously during infusion. d. Have the prescription, dosage calculations, and pump settings checked by a second nurse.

d Accidental overdose can cause death. Another nurse should verify accuracy of the prescription, dose, and pump settings to prevent harm to the client.

A client is admitted to the cardiac care unit with a myocardial infarction. The cardiac monitor reveals a series of premature ventricular complexes (PVCs). The nurse anticipates that the client will be receiving a prescription for: a. Atropine (Atropine Sulfate) b. Epinephrine (Adrenaline) c. Sodium bicarbonate (Sodium bicarbonate 5% injection) d. Amiodarone (Cordarone)

d Amiodarone suppresses ventricular activity; therefore, it is used for treatment of PVCs. It works directly on the heart tissue and slows the nerve impulses in the heart.

A client who has been diagnosed with a myocardial infarction is receiving morphine for pain. The client takes digoxin (Lanoxin) and fluoxetine (Prozac) at home. The client asks why a stool softener has been added to the medication regimen. The nurse explains the importance of not straining when having a bowel movement and that what has put the client at risk for constipation? a. Digoxin b. Poor peripheral perfusion c. Fluoxetine d. Opioid use

d Opioids decrease peristalsis, which may precipitate constipation; straining at stool should be avoided to prevent the Valsalva maneuver, which increases demands on the heart.

A client is admitted to the cardiac care unit with an anterior lateral myocardial infarction. The health care provider prescribes 500 mL of D5W with 50 mg of nitroglycerin to be administered intravenously to relieve pain. The nurse should assess for which most common side effect of this medication? a. Nausea b. Syncope c. Bradycardia d. Hypotension

d The major action of intravenous nitroglycerin (Nitrostat IV) is venous and then arterial dilation, leading to a decrease in blood pressure.

A health care provider prescribes tissue plasminogen activator (t-PA) to be administered intravenously over one hour for a client experiencing a myocardial infarction. The nursing priority that is specific to this medication is the assessment of the client's: a. Respiratory rate b. Peripheral pulses c. Level of consciousness d. Intravenous insertion site

d The most common adverse effect of a tissue plasminogen activator is bleeding because of the thrombolytic action of the drug.

Which data indicates to the nurse that simvastatin (Zocor), an HMG-CoA reductase inhibitor, is effective? 1. The client's blood pressure is 132/80. 2. The client's cholesterol level is 180 mg/dL. 3. The client's LDL is 180 mg/dL. 4. The client's HDL is 35 mg/dL.

2 A cholesterol level less than 200 mg/dL is desirable and indicates the medica- tion is effective.

The client diagnosed with angina is prescribed nitroglycerin (Nitrobid) and tells the nurse, "I don't understand why I can't take my Viagra. I need to take it so that I can make love to my wife." Which statement is the nurse's best response? 1. "If you take the medications together, you may get very low blood pressure." 2. "You are worried your wife will be concerned if you cannot make love." 3. "If you wait at least 8 hours after taking your nitroglycerin (NTG), you can take your Viagra." 4. "You should get clarification with your HCP about your taking Viagra."

1 Life-threatening hypotension can result with concurrent use of nitroglycerin and sildenafil (Viagra).

13. The client with coronary artery disease is prescribed cholestyramine, a bile-acid sequestrant. Which intervention should the nurse implement when administering the medication? 1. Administer the medication with fruit juice. 2. Instruct the client to decrease fiber when taking the medication. 3. Monitor the cholesterol level before giving medication. 4. Assess the client for upper-abdominal discomfort.

1 This medication should be administered with water, fruit juice, soup, or pulpy fruit (applesauce, pineapple) to reduce the risk of esophageal irritation and impintervention.

The nurse is administering 0.5 inch of nitropaste, a coronary vasodilator. How much paste should the nurse apply to the application paper? 1. A 2. B 3. C 4. D

1. A. The line is in increments of 0.5 (1/2 inch) and the order is 0.5 inch, or 1/2 inch; therefore, the nurse should apply this much paste.

20. The nurse is preparing to administer a nitroglycerin patch to a client diagnosed with coronary artery disease. Which interventions should the nurse implement? Rank in order of performance. 1. Date and time the nitroglycerin patch. 2. Remove the old patch. 3. Clean the site of the old patch. 4. Apply the nitroglycerin patch. 5. Check the patch against the MAR.

5,1,2,4,3

What is the most significant adverse effect of spironolactone? A Hyperkalemia B Hypokalemia C Hypernatremia D Hyponatremia

A

Which ACE inhibitor causes angioedema? A Benazepril B Captopril C Enalapril D Lisonopril

A

Which loop diuretic causes hives and tinnitus? A Bumetanide B Ferusemide

A

Which of the following medications will decrease the therapeutic effects of quinapril (Accupril)? A. NSAIDs B. Allopurinol C. Antacids D. All of the above

A

A nurse is caring for a patient who is taking an angiotensin-converting enzyme inhibitor and develops a dry, nonproductive cough. What is the nurse's priority action? A Call the health care provider to switch the medication. B Assess the patient for other symptoms of upper respiratory infection. C Instruct the patient to take antitussive medication until the symptoms subside. D Tell the patient that the cough will subside in a few days.

A Angiotensin-converting enzyme inhibitors prevent the breakdown of bradykinin, frequently causing a nonproductive cough. Angiotensin receptor blocking agents do not block this breakdown, thus minimizing this annoying side effect. The patient should be switched to a different medication if the side effect cannot be tolerated.

What statement indicates to the nurse that the patient needs additional instruction about antihypertensive treatment? A "I will check my blood pressure daily and take my medication when it is over 140/90." B "I will include rest periods during the day to help me tolerate the fatigue my medicine may cause." C "I will change my position slowly to prevent feeling dizzy." D "I will not mow my lawn until I see how this medication makes me feel."

A Antihypertensive medications need to be taken routinely to maintain a normotensive state and prevent occurrence of complications. Many patients do not adhere to this regimen because hypertension itself does not cause symptoms, whereas the medication can cause some untoward effects. Patient teaching is essential. If the patient indicates that he will take rest periods and change positions slowly to avoid orthostatic hypotension, he is demonstrating compliance with the treatment regimen.

A nurse is providing instructions to a client who is on nicotinic acid for the treatment of hyperlipidemia. Which statement made by the nurse indicates a comprehension of the instructions? A "I should take aspirin 30 minutes before nicotinic acid". B "I will drink alcohol in moderation". C "Yellowing of the skin is a common side effect". D "This medication is taken on an empty stomach".

A The use of aspirin or a nonsteroidal anti-inflammatory drug 30 minutes before decreases flushing which is a side effect of taking nicotinic acid.

How do thiazide diuretics lower blood pressure? A By acting on arterioles B By blocking beta1 receptors C By blocking alpha1 receptors D By relaxing venous smooth muscle

A Thiazide diuretics cause vasodilation by acting directly on arterioles, which lowers blood pressure.

A patient is ordered furosemide (Lasix) to be given via intravenous push. Which interventions will the nurse perform? (Select all that apply.) A Administer at a rate no faster than 20 mg/min. B Assess lung sounds before and after administration. C Assess blood pressure before and after administration. D Maintain accurate intake and output record. E Monitor ECG continuously. F Insert an arterial line for continuous blood pressure monitoring.

A B C D Furosemide (Lasix) can be infused via intravenous push at the rate of 20 mg/min. Furosemide is a diuretic and will decrease fluid in alveoli, and assessing lung sounds can help to determine therapeutic effect. Blood pressure should decrease with the administration of a diuretic. It is appropriate to monitor before and after administration. It is appropriate to monitor intake and output for a patient receiving a diuretic. There is no need to insert an arterial line to continuously monitor the blood pressure since it should not fluctuate that dramatically. Also, there is no need to continuously monitor ECG since the medication is not cardiotoxic.

The function of HDL is to remove cholesterol from the blood stream and deliver it to the liver for excretion in the bike. What is the desired rate? A less than 60 B greater than 60 C between 50 and 60 D between 35 and 50

B Greater than 60 is desirable. Less than 60 is but. It less than 35 is a low to moderate risk. Below 35 is high risk

The nurse is listening to a client's heartbeat and is focusing on the second heart sound. Which heart valves produce this sound? a. Aortic and tricuspid b. Aortic and pulmonic c. Mitral and pulmonic d. Mitral and tricuspid

B

The three primary elements that determine the pressure in the cardiovascular system are heart rate, stroke volume, and __________. A. Baroreceptors B. Peripheral resistance C. Renin-Angiotensin-Aldosterone System (RAAS) D. All of the above

B

What enzyme is inhibited by the use of statin drugs? A hsCRP B HMG CoA C VLDL D Cholesterol

B

Question 7 What is the first intervention for a client experiencing MI? A Administer morphine B Administer oxygen C Administer sublingual nitroglycerin D Obtain an ECG

B Administering supplemental oxygen to the client is the first priority of care. The myocardium is deprived of oxygen during an infarction, so additional oxygen is administered to assist in oxygenation and prevent further damage. Morphine and nitro are also used to treat MI, but they're more commonly administered after the oxygen. An ECG is the most common diagnostic tool used to evaluate MI.

A student nurse was asked to take the blood pressure of a patient in the emergency room. The BP reads 140/90 mmHg. She would be correct to say that this BP reading belongs to which classification? A. Normal B. Prehypertension C. Class I Hypertension D. Class II Hypertension

C

The nurse acknowledges that beta blockers are as effective as antianginals because they do what? a. Increase oxygen to the systemic circulation. b. Maintain heart rate and blood pressure. c. Decrease heart rate and decrease myocardial contractility. d. Decrease heart rate and increase myocardial contractility.

C

A patient is taking pravastatin sodium (Pravachol). Which assessment finding requires immediate action by the nurse? A Headache B Slight nausea C Muscle pain D Fatigue

C Patients who experience severe muscle pain while taking pravastatin sodium (Pravachol) need to report the findings right away, as this may be indicative of rhabdomyolysis, a muscle disintegration that can become fatal.

A patient taking spironolactone (Aldactone) has been taught about the medication. Which menu selection indicates that the patient understands teaching related to this medication? A Potatoes B Lima beans C Chicken D Strawberries

C Spironolactone is a potassium-sparing diuretic that could potentially cause hyperkalemia. Chicken is the only appropriate choice of the foods listed because it is lower in potassium. Potatoes, lima beans, and strawberries are all known to contain high levels of potassium.

Untreated hypertension can result to _________. A. Stroke B. Renal failure c. Loss of vision D. All of the above

D

Which statement made by the client demonstrates a need for further instruction regarding the use of nitroglycerin? a. "If I get a headache, I should keep taking nitroglycerin and use Tylenol for pain relief." b. "I should keep my nitroglycerin in a cool, dry place." c. "I should change positions slowly to avoid getting dizzy." d. "I can take up to five tablets at 3-minute intervals for chest pain if necessary."

D

After reviewing the client's chart upon admission to the unit, the nurse consults the health care provider about a new order for lovastatin (Mevacor). What triggered the nurse's action? A Blood glucose of 182 mg/dL B History of peptic ulcers C History of high cholesterol D Elevated liver enzymes

D Treatment with any of the statins for elevated cholesterol and low-density lipoprotein (LDL) levels is contraindicated for clients with active liver disease because these agents can cause increases in liver function.

A client's serum digoxin level is drawn, and it is 0.4 ng/mL. What is the nurse's priority action? a. Administer ordered dose of digoxin. b. Hold future digoxin doses. c. Administer potassium. d. Call the health care provider.

a

A nurse is caring for a client who is taking an angiotensin-converting enzyme inhibitor and develops a dry, nonproductive cough. What is the nurse's priority action? a. Call the health care provider to switch the medication. b. Assess the client for other symptoms of upper respiratory infection. c. Instruct the client to take antitussive medication until the symptoms subside. d. Tell the client that the cough will subside in a few days

a

The nurse explains that which beta blocker category is preferred for treating hypertension? a. Beta1 blocker b. Beta2 blocker c. Beta1 and beta2 blockers d. Beta2 and beta3 blockers

a

Which assessment finding will alert the nurse to suspect early digoxin toxicity? a. Loss of appetite with slight bradycardia b. Blood pressure 90/60 mm Hg c. Heart rate 110 beats per minute d. Confusion and diarrhea

a

Which assessment indicates a therapeutic effect of mannitol (Osmitrol)? a. Decreased intracranial pressure b. Decreased potassium c. Increased urine osmolality d. Decreased serum osmolality

a

A client's cardiac monitor indicates ventricular tachycardia. The nurse assesses the client and identifies an increase in apical pulse rate from 100 to 150 beats per minute. An appropriate treatment plan includes: a. Amiodarone bolus b. Intracardiac epinephrine c. Insertion of a pacemaker d. Cardiopulmonary resuscitation (CPR)

a Amiodarone suppresses ventricular activity; therefore, it is used for treatment of premature ventricular complexes (PVCs). It works directly on the heart tissue and slows the nerve impulses in the heart.

A nurse identifies signs of electrolyte depletion in a client with heart failure who is receiving bumetanide (Bumex) and digoxin (Lanoxin). What does the nurse determine is the cause of the depletion? a. Diuretic therapy b. Sodium restriction c. Continuous dyspnea d. Inadequate oral intake

a Diuretic therapy that affects the loop of Henle generally involves the use of drugs (e.g., bumetanide) that directly or indirectly increase urinary sodium, chloride, and potassium excretion.

A calcium channel blocker has been ordered for a client. Which condition in the client's history is a contraindication to this medication? a. Hypokalemia b. Dysrhythmias c. Hypotension d. Increased intracranial pressure

c

A client who had a myocardial infarction is in the coronary care unit on a cardiac monitor. The nurse observes ventricular irritability on the screen. What medication should the nurse prepare to administer? a. Digoxin (Lanoxin) b. Furosemide (Lasix) c. Amiodarone (Cordarone) d. Norepinephrine (Levophed)

c Amiodarone decreases the irritability of the ventricles by prolonging the duration of the action potential and refractory period. It is used in the treatment of ventricular dysrhythmias.


Related study sets

Entrepreneurial Small Business Chapter 4 - Creativity, Opportunity, and Feasibility

View Set

NUR313 Ch. 14 Communication in the Nurse-Patient Relationship

View Set

appendicular skeleton lab assignment

View Set

NBDHE Practice Test MEGA-SET 💪🏻

View Set

Corporate Finance for Managers Chapter 15 Study Guide

View Set